++
++
++
++
You are appointed as a cardiac epidemiologist advisor on an international committee setting global health policy. Your committee is assigned to predict health trends in a Pacific East Asian nation with a population and environment very similar to China. At this point in history, the nation is just moving from the first classic stage in the epidemiologic transition (“pestilence and famine”) to the second classic stage (“receding pandemics”). In regard to the anticipated patterns of cardiovascular disease (CVD) in this nation, which of the following is true?
++
++
++
A. As this nation enters the stage of receding pandemics, you would expect the majority of CVD morbidity and mortality to be due to cardiomyopathies secondary to infectious agents.
++
++
B. Each nation or geographical location progresses through the five stages of epidemiologic transition identically in regard to CVD risk.
++
++
C. In this nation, when CVD mortality peaks, you would expect stroke mortality to be greater than coronary heart disease mortality.
++
++
D. One would expect a very homogeneous epidemiologic pattern of CVD risk throughout this nation over time.
++
++
E. You anticipate that CVD mortality will inexorably climb as this nation progresses through the five stages of the epidemiologic transition.
+
++
The answer is C. (Chap. 266e) The global rise in cardiovascular disease (CVD) is the result of an unprecedented transformation in the causes of morbidity and mortality during the 20th century. Known as the epidemiologic transition, this shift is driven by industrialization, urbanization, and associated lifestyle changes and is taking place in every part of the world among all races, ethnic groups, and cultures. The transition is divided into four basic stages: pestilence and famine, receding pandemics, degenerative and man-made diseases, and delayed degenerative diseases. A fifth stage, characterized by an epidemic of inactivity and obesity, is emerging in some countries. The stages of the epidemiologic transition are shown in Table V-1.
++
++
Note that CVD mortality peaks in the third stage and then tends to decline aided by preventive strategies such as smoking cessation programs and effective blood pressure control, acute hospital management, and technologic advances, such as the availability of bypass surgery. In the stage of receding pandemics, the majority of CVD risk tends to be due to coronary heart disease, stroke, and rheumatic heart disease. Although the stages of epidemiologic transition is a useful construct to generalize expectations regarding CVD epidemiology over time, every nation experiences these stages in unique fashion due to environmental, behavioral, and genetic differences. For example, in China and Japan, stroke causes more deaths than coronary heart disease in a ratio of about three to one, as we would expect for this hypothetical nation.
++
++
++
You are working in a rural health clinic in Northern India. You evaluate an 8-year-old boy who has never seen a physician. His mother tells you that he is unable to keep up with his peers in terms of physical activity. On your initial examination of his skin, you notice clubbing and cyanosis in his feet, but his hands appear normal. Without any further examination, you suspect that he has which of the following congenital abnormalities?
++
++
++
++
++
B. Dextro-transposition of the great arteries (TGA)
++
++
C. Patent ductus arteriosus with secondary pulmonary hypertension
++
++
++
++
E. Ventricular septal defect
+
++
The answer is C. (Chap. 267) This patient has classic findings of differential cyanosis, or isolated cyanosis (and clubbing in this case) of the lower, but not upper, extremities. This limits the differential diagnosis to only one possibility: patent ductus arteriosus with secondary pulmonary hypertension causing right-to-left shunting at the level of the great vessels, distal to the upper extremity branches. Tetralogy of Fallot often causes central cyanosis due to right-to-left intracardiac shunting with no differential between the upper and lower extremities. Ventricular septal defect (VSD) and atrial septal defect (ASD) often have no cyanosis evident as they are most often left-to-right shunts (unless pulmonary hypertension develops resulting in Eisenmenger syndrome). Transposition of the great arteries (TGA) may exhibit varying degrees of central cyanosis, depending on the intracardiac anatomy (most often present with an ASD).
++
++
++
A 55-year-old African American woman with no past medical history presents precipitously to the emergency department with syncope and the electrocardiogram (ECG) noted in Figure V-3A below. You also note painful subcutaneous nodules on her legs (similar to those pictured in Figure V-3B below). You suspect that the cardiac and skin abnormalities are most likely due to:
+
+
+
++
FIGURE V-3 Courtesy of Robert Swerlick, MD; with permission.
+
++
++
++
++
++
++
++
++
++
D. Systemic lupus erythematosus
++
++
+
++
The answer is C. (Chap. 267) The electrocardiogram (ECG) shows complete heart block, and the leg lesions are consistent with erythema nodosum. This combination of findings is most likely due to sarcoidosis. Extrapulmonary sarcoidosis may manifest as cardiac conduction abnormalities (most common at the atrioventricular [AV] node), skin and joint lesions, hypercalcemia, and neurologic deficits. Hypothyroidism may cause sinus bradycardia but is usually not a cause of complete heart block. Also, the lower extremity skin finding of hypothyroidism is myxedema, a doughy, nonpitting, nonpainful edema. Carney syndrome is characterized by multiple lentigines and atrial myxomata. Systemic lupus erythematosus can be associated by erythema nodosum, although it does not often cause conduction system disease in adults. However, neonatal lupus is a common cause of complete heart block.
++
++
++
You are evaluating Mr. Estebez, a 67-year-old owner of a wildly successful chain of sushi restaurants. He complains of shortness of breath with exertion, lower extremity edema, and awakening at night feeling acutely short of breath. You wish to assess his volume status, and know that jugular venous pulse (JVP) assessment is the single most important physical examination measurement to aid you in this component of your evaluation. Which of the following statements regarding JVP measurement is true?
++
++
++
A. If done properly, the angle of inclination matters little to the measurement of JVP.
++
++
B. In normal patients, the JVP rises with inspiration due to the augmented volume loading of the right heart.
++
++
C. Measurement of the elevation of the top of the JVP and the sternal inflection point (angle of Louis) will provide a highly accurate measurement of central venous pressure.
++
++
D. The external jugular is preferred over the internal jugular vein due to its ease of visibility.
++
++
E. Venous pulsations above the clavicle in the sitting position are abnormal.
+
++
The answer is E. (Chap. 267) Jugular venous pulse (JVP) is the single most important physical examination measurement from which to estimate a patient’s volume status. The internal jugular vein is preferred because the external jugular vein is valved and not directly in line with the superior vena cava and right atrium. Precise estimation of the central venous or right atrial pressure from bedside assessment of the jugular venous waveform has proved difficult. Venous pressure traditionally has been measured as the vertical distance between the top of the jugular venous pulsation and the sternal inflection point (angle of Louis). A distance >4.5 cm at 30 degrees of elevation is considered abnormal. However, the actual distance between the mid-right atrium and the angle of Louis varies considerably as a function of both body size and the patient angle at which the assessment is made (30, 45, or 60 degrees). The use of the sternal angle as a reference point leads to systematic underestimation of central venous pressure (CVP), and this method should be used less for semi-quantification than to distinguish a normal from an abnormally elevated CVP. Venous pulsations above the right clavicle in the sitting position are clearly abnormal, as the distance between the clavicle and the right atrium is at least 10 cm. Normally, the venous pressure should fall by at least 3 mmHg with inspiration. Kussmaul sign is defined by either a rise or a lack of fall of the JVP with inspiration and is classically associated with constrictive pericarditis, although it has been reported in patients with restrictive cardiomyopathy, massive pulmonary embolism, right ventricular infarction, and advanced left ventricular systolic heart failure. It is also a common, isolated finding in patients after cardiac surgery without other hemodynamic abnormalities.
++
++
++
Which of the following statements regarding blood pressure measurements is true?
++
++
++
A. Systolic pressure increases and diastolic pressure decreases when measured in more distal arteries.
++
++
B. Systolic leg blood pressures are usually as much as 20 mmHg lower than arm blood pressures.
++
++
C. The concept of “white coat hypertension” (blood pressures measured in office or hospital settings significantly higher than in nonclinical settings) has been shown to be a myth.
++
++
D. The difference in blood pressure measured in both arms should be less than 20mmHg.
++
++
E. Using a blood pressure cuff that is too small will result in a marked underestimation of the true blood pressure.
+
++
The answer is A. (Chap. 267) The length and width of the blood pressure cuff bladder should be 80% and 40% of the arm’s circumference, respectively. A common source of error in practice is to use an inappropriately small cuff, resulting in marked overestimation of true blood pressure, or an inappropriately large cuff, resulting in underestimation of true blood pressure. The cuff should be inflated to 30 mmHg above the expected systolic pressure and the pressure released at a rate of 2–3 mmHg/sec. Systolic and diastolic pressures are defined by the first and fifth Korotkoff sounds, respectively. Very low (even 0 mmHg) diastolic blood pressures may be recorded in patients with chronic, severe aortic regurgitation (AR) or a large arteriovenous fistula because of enhanced diastolic “run-off.” In these instances, both the phase IV and phase V Korotkoff sounds should be recorded. Blood pressure is best assessed at the brachial artery level, although it can be measured at the radial, popliteal, or pedal pulse level. In general, measured systolic pressure increases and diastolic pressure decreases when measured in more distal arteries. Blood pressure should be measured in both arms, and the difference should be less than 10 mmHg. A blood pressure differential that exceeds this threshold may be associated with atherosclerotic or inflammatory subclavian artery disease, supravalvular aortic stenosis, aortic coarctation, or aortic dissection. Systolic leg pressures are usually as much as 20 mmHg higher than systolic arm pressures. Greater leg–arm pressure differences are seen in patients with chronic severe AR as well as patients with extensive and calcified lower extremity peripheral arterial disease. The ankle-brachial index (lower pressure in the dorsalis pedis or posterior tibial artery divided by the higher of the two brachial artery pressures) is a powerful predictor of long-term cardiovascular mortality. The blood pressure measured in an office or hospital setting may not accurately reflect the pressure in other venues. “White coat hypertension” is defined by at least three separate clinic-based measurements >140/90 mmHg and at least two non–clinic-based measurements <140/90 mmHg in the absence of any evidence of target organ damage. Individuals with white coat hypertension may not benefit from drug therapy.
++
++
++
A 75-year-old man presents to your emergency department appearing quite ill. His family says he has not had his normal energy for the last 6 months, and they noted he was confused and lethargic for the last day or two. As you take a history from the family, you palpate the patient’s radial pulse and notice a regular beat-to-beat variability of pulse amplitude, although his rhythm is regular. Indeed, as you later take his blood pressure, you note that only every other phase I (systolic) Korotkoff sound is audible as the cuff pressure is slowly lowered and that this is independent of the respiratory cycle. Based on this, you suspect this patient has which of the following?
++
++
++
++
++
++
++
C. Constrictive pericarditis
++
++
++
++
E. Severe left ventricular dysfunction
+
++
The answer is E. (Chap. 267) This patient has evidence of pulsus alternans. Pulsus alternans is defined by beat-to-beat variability of pulse amplitude. It is present only when every other phase I Korotkoff sound is audible as the cuff pressure is lowered slowly, typically in a patient with a regular heart rhythm and independent of the respiratory cycle. Pulsus alternans is seen in patients with severe left ventricular systolic dysfunction and is thought to be due to cyclic changes in intracellular calcium and action potential duration. When pulsus alternans is associated with electrocardiographic T-wave alternans, the risk for an arrhythmic event appears to be increased. Cardiac tamponade or large pericardial effusions can be associated with electrical alternans (a regular variability in the QRS voltage or vector) or pulsus paradoxus, a difference between the systolic pressure at which the Korotkoff sounds are first heard (during expiration) and the systolic pressure at which the Korotkoff sounds are heard with each heartbeat, independent of the respiratory phase. This is an exaggerated consequence of interventricular dependence. Atrial fibrillation would result in beat-to-beat variability of the pulse amplitude but also an irregularly irregular rhythm.
++
++
++
A 78-year-old man is admitted to the intensive care unit with decompensated heart failure. He has longstanding ischemic cardiomyopathy. ECG shows atrial fibrillation and left bundle branch block. Chest radiograph shows cardiomegaly and bilateral alveolar infiltrates with Kerley B lines. Which of the following is least likely to be present on physical examination?
++
++
++
++
++
++
++
++
++
D. Reversed splitting of the second heart sound
++
++
+
++
The answer is A. (Chap. 267) A fourth heart sound indicated left ventricular presystolic expansion and is common among patients in whom active atrial contraction is important for ventricular filling. A fourth heart sound is not found in atrial fibrillation. An irregular heart rate is characteristic of atrial fibrillation. The irregular rate is often characterized as “irregularly irregular.” A third heart sound occurs during the rapid filling phase of ventricular diastole and indicates heart failure. Reversed splitting of the second heart sound occurs with left bundle branch block, as this patient demonstrates. Kussmaul sign is defined by either a rise or a lack of fall of the JVP with inspiration and is classically associated with constrictive pericarditis, although it has been reported in patients with restrictive cardiomyopathy, massive pulmonary embolism, right ventricular infarction, and advanced left ventricular systolic heart failure. It is also a common, isolated finding in patients after cardiac surgery without other hemodynamic abnormalities.
++
++
++
A 24-year-old man is referred to cardiologist after an episode of syncope while playing basketball. He has no recollection of the event, but he was told that he collapsed while running. He awakened lying on the ground and suffered multiple contusions as a result of the fall. He has always been an active individual but recently has developed some chest pain with exertion that has caused him to restrict his activity. His father died at age 44 while rock climbing. He believes his father’s cause of death was sudden cardiac death and recalls being told his father had an enlarged heart. On examination, the patient has a III/VI mid-systolic crescendo-decrescendo murmur. His ECG shows evidence of left ventricular hypertrophy. You suspect hypertrophic cardiomyopathy as the cause of the patient’s heart disease. Which of the following maneuvers would be expected to cause an increase in the loudness of the murmur?
++
++
++
++
++
++
++
++
++
++
++
++
++
+
++
The answer is F. (Chap. 267) When a murmur of uncertain cause is identified on physical examination, a variety of physiologic maneuvers can be used to assist in the elucidation of the cause. Commonly used physiologic maneuvers include change with respiration, Valsalva maneuver, position, and exercise. In hypertrophic cardiomyopathy, there is asymmetric hypertrophy of the interventricular septum, which creates a dynamic outflow obstruction. Maneuvers that decrease left ventricular filling will cause an increase in the intensity of the murmur, whereas those that increase left ventricular filling will cause a decrease in the murmur. Of the interventions listed, both standing and a Valsalva maneuver will decrease venous return and subsequently decrease left ventricular filling, resulting in an increase in the loudness of the murmur of hypertrophic cardiomyopathy. Alternatively, squatting increases venous return and thus decreases the murmur. Maximum handgrip exercise also results in a decreased loudness of the murmur.
++
++
++
A 75-year-old woman with widely metastatic non–small-cell lung cancer is admitted to the intensive care unit with a systolic blood pressure of 73/52 mmHg. She presented complaining of fatigue and worsening dyspnea over the last 3–5 days. Her physical examination shows elevated neck veins. Chest radiograph shows a massive, water bottle–shaped heart shadow and no new pulmonary infiltrates. Which of the following additional findings is most likely present on physical examination?
++
++
++
A. Fall in systolic blood pressure >10 mmHg with inspiration
++
++
B. Lack of fall of the jugular venous pressure with inspiration
++
++
C. Late diastolic murmur with opening snap
++
++
D. Pulsus parvus et tardus
++
++
E. Rapid y-descent of jugular venous pressure tracing
+
++
The answer is A. (Chap. 267) The patient is likely to have pericardial tamponade from metastatic cancer as suggested by her elevated neck veins, heart shadow shape and size, and predisposing condition. Because of the exaggerated interventricular dependence, the normal (<10 mmHg) fall in systemic blood pressure with inspiration is exaggerated (often >15 mmHg) with cardiac tamponade. This is referred to pulsus paradoxus, although it is in fact an augmentation of a normal finding. Kussmaul sign, or a lack of fall of the jugular venous pressure with inspiration, is a sign usually denoting a lack of compliance in the right ventricle, as seen most frequently in constrictive pericarditis, although it may be found in restrictive cardiomyopathy or massive pulmonary embolism. A slow y-descent, which follows the peak of the v wave, of jugular venous pressure tracing is indicative of cardiac tamponade as the restricted ventricle is slow to fill during diastole. The x-descent (systolic) may be augmented because ventricular ejection results in relative unloading of the intrapericardial pressure. Pulsus parvus et tardus, or small and slow arterial pulsation, is a late finding in aortic stenosis. Late diastolic murmur with opening snap is found in mitral stenosis.
++
++
++
Which of the following statements regarding normal depolarization patterns of the heart is true?
++
++
++
A. Each normal sinus beat is initiated by spontaneous depolarization in the atrioventricular (AV) node.
++
++
B. The normal order of depolarization is as follows: sinoatrial (SA) node – atrial myocardium – AV node – His bundle – Purkinje fibers – ventricular myocardium.
++
++
C. The right bundle branch bifurcates into an anterior and posterior fascicle.
++
++
D. The SA node is unique in its ability to spontaneously depolarize, a quality known as automaticity.
++
++
E. Within the ventricular myocardium, depolarization sweeps from epicardium to endocardium.
+
++
The answer is B. (Chap. 268) The depolarization stimulus for the normal heartbeat originates in the sinoatrial (SA) node, or sinus node, a collection of pacemaker cells. These cells fire spontaneously; that is, they exhibit automaticity. Other cells exhibit automaticity, although at a slower rate, including AV nodal (junctional) cells and the Purkinje fiber cells. The first phase of cardiac electrical activation is the spread of the depolarization wave through the right and left atria, followed by atrial contraction. Next, the impulse stimulates pacemaker and specialized conduction tissues in the AV nodal and His bundle areas; together, these two regions constitute the AV junction. The bundle of His bifurcates into two main branches, the right and left bundles, which rapidly transmit depolarization wavefronts to the right and left ventricular myocardium by way of Purkinje fibers. The main left bundle bifurcates into two primary subdivisions: a left anterior fascicle and a left posterior fascicle. The depolarization wavefronts then spread through the ventricular wall, from endocardium to epicardium, triggering ventricular contraction.
++
++
++
All of the following ECG waveforms are matched correctly to the cardiac cycle that they represent EXCEPT:
++
++
++
A. P wave – atrial repolarization
++
++
B. PR interval – atrial repolarization
++
++
C. QRS complex – ventricular depolarization
++
++
D. T wave – ventricular repolarization
++
++
E. U wave – ventricular repolarization
+
++
The answer is A. (Chap. 268) The ECG waveforms are labeled alphabetically, beginning with the P wave, which represents atrial depolarization. The QRS complex represents ventricular depolarization, and the ST-T-U complex (ST segment, T wave, and U wave) represents ventricular repolarization. The J point is the junction between the end of the QRS complex and the beginning of the ST segment. Atrial repolarization (STa and Ta) is usually too low in amplitude to be detected, but it may become apparent in conditions such as acute pericarditis and atrial infarction.
++
++
++
Which of the following mean QRS vectors in the frontal electrocardiographic plane is matched appropriately to its designation?
++
++
++
A. –20 degrees – normal axis
++
++
B. –35 degrees – right axis deviation
++
++
C. –110 degrees – left axis deviation
++
++
D. –80 degrees – extreme axis deviation
++
++
E. All of the above are incorrect.
+
++
The answer is A. (Chap. 268) Figure V-12 depicts the limb leads and respective frontal plane axis categories on the ECG. Note that normal axis extends from –30 to 90 or 100 degrees.
+
+
++
+
++
++
++
Which of the following is represented in the ECG shown in Figure V-13?
+
+
++
FIGURE V-13 From Fuster V, Walsh R, Harrington R, et al (eds): Hurst’s The Heart, 13th ed. New York, NY: McGraw-Hill, 2011, Fig. 15-31B.
+
++
++
++
A. First-degree heart block
++
++
B. Left bundle branch block
++
++
++
++
D. Right bundle branch block
++
++
E. S1Q3T3 (McGinn-White pattern) indicative of right ventricular strain
+
++
The answer is D. (Chap. 268) This ECG exhibits classic findings of a right bundle branch block with an rSR’ in V1 and broad-based terminal S wave in I, II, and V6. Left bundle branch block would have a similarly wide QRS, but with the terminal deflections on the QRS occurring with a leftward and posterior vector leading to a large R pattern in I and V6 and a QS pattern in V1. S1Q3T3 is a nonspecific ECG finding for right heart strain and is not present in this ECG. P-pulmonale is a term used to indicate right atrial abnormality or enlargement, seen on ECG as a tall P wave in lead II or V1 (>2.5 mm). First-degree heart block is seen as a PR interval >200 msec (one large box on a standard-speed ECG).
++
++
++
Which of the following options describes the primary finding in the ECG shown in Figure V-14?
+
+
++
+
++
++
++
A. Left ventricular hypertrophy
++
++
++
++
C. Peaked T waves, possibly hyperkalemia
++
++
++
++
E. ST elevation in the anterior precordial leads; suspect anterior myocardial ischemia
+
++
The answer is B. (Chap. 268) This is a normal 12-lead ECG. The heart rate is about 75 bpm. The P-wave morphology and axis are normal, indicating normal atrial size and activation. The QRS axis is 70 degrees, which is normal. QRS duration is 0.08 seconds (normal), and QT interval 0.36 seconds (normal). There is no abnormal ST elevation or depression, and the T-wave morphology and size are normal.
++
++
++
A 56-year-old construction worker with hypertension and a prior history of tobacco abuse presents to the emergency department with 30 minutes of acute-onset nausea, dyspnea, and chest pressure. His initial ECG is presented in Figure V-15. All of the following are present in this ECG EXCEPT:
+
+
++
+
++
++
++
A. Inferior myocardial ischemia
++
++
++
++
C. Posterior myocardial ischemia
++
++
++
++
E. Ventricular tachycardia
+
++
The answer is E. (Chap. 269e) Mr. Wilson is suffering an acute inferolateral myocardial infarction (MI). Note the ST elevations in the inferior leads (II, III, aVF) and in V6 (lateral precordial lead). Also, the striking ST depressions in the anterior precordial leads (V1–V4) are indicative of posterior ischemia. Here, the presence of ST depressions in an anterior lead represents the “mirror” of ST elevations in a posterior location. One can visualize the ECG pattern in the anterior leads upside down to see the reciprocal nature of the ST depressions. Although on initial glance, the QRS pattern appears wide in the anterior leads suggesting ventricular tachycardia, on further examination, there are P waves present that are associated with each QRS (seen most clearly in leads II and V1); thus, the rhythm is sinus tachycardia. The wide appearance of the QRS is due to the striking ST deviation.
++
++
++
A 56-year-old construction worker with hypertension and a prior history of tobacco abuse presents to the emergency department with 30 minutes of acute-onset nausea, dyspnea, and chest pressure. His initial ECG is presented in Figure V-15. In the patient described, which of the following coronary arteries is most likely occluded?
+
+
++
+
++
++
++
A. First septal perforator
++
++
B. Proximal left anterior descending artery
++
++
C. Proximal left circumflex artery
++
++
D. Proximal left anterior descending and left circumflex arteries
++
++
+
++
The answer is E. (Chap. 269e) This patient has an acute inferior myocardial infarction.
++
Although it is possible that the left circumflex (LCx) arteries has coronary disease, the combination of a greater degree of ST segment elevation in lead III when compared to lead II, and the absence of ST elevation in the lateral leads (I and aVL) both make the right coronary artery (RCA) the culprit artery. The ST depressions in leads V1 through V3 indicate posterior ST elevation myocardial infarction from the occluded RCA. To further localize the lesion electrocardiographically, one should next obtain a right sided ECG, wherein the precordial leads are flipped to the right side of the chest, simply mirroring their usual left-sided position. The presence of >0.5mm of ST segment elevation in V4R would indicate a proximal right coronary artery occlusion, which carries a much higher mortality than a distal RCA occlusion.
++
++
++
A 48-year-old woman visits you in primary care clinic for initial evaluation after moving across the country. You have no past medical records, although she insists they were mailed a week prior. She states that she has had “some heart troubles,” but she is not clear on the details. Also, she is on pills for “cholesterol and blood pressure.” The initial ECG is shown in Figure V-17. Which of the following statements regarding this ECG is true?
+
+
++
+
++
++
++
A. It is likely she has suffered a prior myocardial infarction.
++
++
B. She is in normal sinus rhythm.
++
++
C. The presence of a left bundle branch block on this ECG indicates dyssynchronous mechanical contraction.
++
++
D. The presence of premature ventricular contractions and tachycardia is concerning for electrolyte imbalance.
++
++
E. The presence of anterior T-wave inversions is concerning in this ECG for acute myocardial ischemia.
+
++
The answer is A. (Chap. 269e) This ECG demonstrates sinus rhythm with premature atrial contractions (not normal sinus rhythm; note the fifth and ninth beats occurring early and preceded by an abnormal P wave indicating atrial origin). Also, the QRS is wide and in a right bundle branch block (RBBB) pattern as indicated by the large terminal R wave in the anterior precordial leads and broad-based terminal S wave in the lateral limb leads (I and aVL). In the presence of an RBBB, T waves in the anterior lead are often inverted and do not indicate acute ischemia. No premature ventricular contractions are present in this ECG. However, Q waves are present in the anterior precordial leads (V1–V3) so that the usual RBBB rsR’ is simply a qR pattern, indicating a prior anterior-septal myocardial infarction.
++
++
++
You are asked to evaluate a 27-year-old internal medicine resident reporting 1 week of cough, coryza, and a low-grade fever. Today, he has developed rapidly escalating chest discomfort while in clinic. He notes that the pain becomes more intense when he takes a deep breath. You perform a standard 12-lead ECG (see Figure V-18). On examination, his blood pressure is normal, he is afebrile, and his jugular venous pulse is not elevated. However, he appears mildly uncomfortable from the chest pain. The next most appropriate step would be which of the following?
+
+
++
+
++
++
++
A. Administer aspirin, intravenous heparin, sublingual nitroglycerin, and clopidogrel.
++
++
B. Emergently obtain transthoracic echocardiogram with possible pericardiocentesis.
++
++
C. Emergently perform coronary angiography to evaluate for acute myocardial infarction.
++
++
D. Prescribe ibuprofen and colchicine.
++
++
E. Refer for treadmill stress test.
+
++
The answer is D. (Chap. 269e) This patient has pericarditis. Often associated temporally with a viral upper respiratory infection, pericarditis is a common noncoronary cause of chest pain. The ECG is confirmatory of the history, showing diffuse, concave ST elevations with PR depressions in most leads. Lead aVR shows PR-segment elevation typical of acute pericarditis. Options A and C would be appropriate choices if the patient were having an acute myocardial infarction. Option E would not be appropriate in the immediate setting of myocardial infarction or pericarditis. Patients with pericarditis without signs of elevated intrapericardial pressure (elevated neck veins, tachycardia, low blood pressure, elevated pulsus paradoxus) or suspicion of purulent pericarditis (fever, sepsis) do not require an echocardiogram (option B). Treatment of acute viral or idiopathic pericarditis is with nonsteroidal anti-inflammatory drugs (NSAIDs) and colchicine.
++
++
++
A large snowstorm hits your area on a Thursday, and the roads are largely impassible until the following Monday. On Monday morning, a 48-year-old man is brought by emergency medical services (EMS) to the emergency department after being found with altered mental status by a neighbor. He is obtunded and unable to provide a history. You note a left brachial artery hemodialysis fistula in place. His initial ECG is shown in Figure V-19. Which of the following electrolyte abnormalities would you expect to find in this patient?
+
+
++
+
++
++
++
++
++
++
++
++
++
++
++
+
++
The answer is B. (Chap. 269e) This patient likely missed a session of hemodialysis due to the impassible roads. In this setting and in combination with the tall, peaked T waves seen on the ECG, one would suspect hyperkalemia. Hyponatremia does not have a stereotypical finding on ECG. Hypercalcemia causes a shortened QT interval (usually due to a shortening specifically of the ST segment with T waves of normal duration). This patient also likely has hypocalcemia evidenced by the prolonged QT interval on ECG. The hypocalcemia in this case is due to hyperphosphatemia secondary to missed hemodialysis. Hypokalemia causes prolonged QT interval, low T-wave voltage, and often the presence of U waves. Hypomagnesemia likewise causes a prolongation of the QT interval and increases the risk of torsades de pointes.
++
++
++
A 66-year-old man is admitted to the hospital for progressive dyspnea on exertion and fatigue. He has a past medical history of tobacco abuse and is widely traveled, recently returning from a multicountry trip through South America. On presentation, his heart rate is 104 bpm and irregularly irregular. Blood pressure is 96/76 mmHg. You note an elevated jugular venous pulsation and marked lower extremity edema. Echocardiogram reveals a left ventricular ejection fraction of 55%, and still images from his echocardiogram are shown in Figure V-20. To elucidate the etiology of his heart failure, which of the following is the most appropriate diagnostic test to perform next?
+
+
++
FIGURE V-20 LA, left atrium; LV, left ventricle; RV, right ventricle.
+
++
++
++
++
++
B. Genetic testing for transthyretin mutations
++
++
C. Positron emission tomography cardiac stress test
++
++
D. Serologies for Trypanosoma cruzi
++
++
E. Serum and urine protein electrophoresis and light chain assay
+
++
The answer is E. (Chap. 271e) This patient demonstrates findings consistent with cardiac amyloidosis. The greatly thickened myocardium with a bright or “sparkly” appearance, left atrial enlargement, and impressive clinical heart failure in the setting of preserved systolic function all suggest infiltrative cardiomyopathy. Strain echocardiographic imaging often reveals decreased myocardial strain and strain rates with relative sparing of the apex in amyloidosis. Atrial fibrillation is common in patients with cardiac amyloidosis and carries a high risk of thromboembolic complications. The preserved ejection fraction renders ischemic disease (as evaluated by positron emission tomography [PET] stress) less likely. Although Trypanosoma cruzi (the etiologic agent of Chagas disease) is endemic in many South American countries, cardiac manifestations include a dilated cardiomyopathy with conduction disease as opposed to a restrictive, thickened myocardium. In the evaluation for cardiac amyloidosis, a reasonable first step is a serologic evaluation for a paraproteinemia with protein electrophoreses of the serum and urine and a light chain assay. Negative results do not rule out amyloidosis, and an endomyocardial biopsy is warranted if clinical suspicion is high enough. If a light chain paraproteinemia is confirmed, a bone marrow biopsy may be warranted later to evaluate for multiple myeloma or a plasma cell dyscrasia. Another common cause of cardiac amyloidosis is deposition of transthyretin (TTR or prealbumin) protein, either in the familial or senile forms. If amyloidosis is confirmed on biopsy and mass spectroscopy cannot be performed or suggests TTR deposition, then genetic testing for known mutations in the TTR gene would be warranted for prognosis and future genetic counseling should be offered to the patient and their family.
++
++
++
Mrs. Jackson is a 45-year-old African American woman with a history of tobacco abuse, breast cancer (status: post mastectomy and radiation), and allergy to shellfish. She presented to your general cardiology clinic 2 weeks prior reporting dyspnea while walking up a hill near her house for the past 6 months. She has never had difficulty with this hill before. Along with the dyspnea, she experiences some vague nausea and breaks out in a sweat. Resting ECG and physical examination are both unrevealing. You referred her for an exercise single-photon emission computed tomography (SPECT) myocardial perfusion technetium-99m (99mTc) sestamibi scan and received the results, pictured in Figure V-21. You quickly assess that her scan shows reversible ischemia. Which arterial territory is most likely involved in the pictured scan?
+
+
++
+
++
++
++
A. Left anterior descending artery
++
++
B. Left circumflex artery
++
++
C. Left main coronary artery
++
++
D. Posterior descending artery
++
++
+
++
The answer is A. (Chap. 271e) This image shows reversible ischemia in all segments of the apex, the distal inferior wall, and the mid to distal septal and inferior walls. This is compatible with ischemic in the distal LAD territory. Ischemia due to significant left main coronary artery stenosis can lead to a “pseudonormal” pattern on nuclear perfusion stress tests, due to balanced ischemia and equally depressed blood flow through the entire left ventricle. Transient dilatation with stress is an important clue to the presence of balanced ischemia and may point toward left main or “triple-vessel” coronary disease. Ischemia of the left circumflex artery would spare the septum and cause reversible ischemia in the lateral and perhaps inferior walls. The right coronary artery and posterior descending artery would present with ischemia of the proximal inferior wall with variable involvement of the inferoseptal segments.
++
++
++
The SA node serves as the dominant pacemaker of the heart in normal sinus rhythm. What property of SA nodal cells allows it to act at the primary pacemaker?
++
++
++
A. Location near the superior lateral area of the right atrium
++
++
B. More numerous intercalated discs of any other myocardial tissue
++
++
C. Most rapid phase 0 depolarization
++
++
D. Only cells with the ability to spontaneously depolarize
++
++
E. Spontaneous depolarization during phase 4 of the action potential at a more rapid rate than any other myocardial cells
+
++
The answer is E. (Chap. 274) Many cells in the heart possess the ability to spontaneously depolarize due to slow spontaneous diastolic depolarization (phase 4). The SA node, AV node, and Purkinje cells all possess this ability. However, in normal physiologic states, the SA node possesses the “steepest” phase 4 slope and thus depolarizes more rapidly than any other potential pacemaker cells. The location does not determine the SA node’s pacemaker ability, and the SA nodal cells have no intercalated discs. Phase 0 is actually slower in nodal cells than in ventricular or atrial nonnodal cells (Figure V-22).
+
+
++
+
++
++
++
Mr. Hendricks is a 21-year-old sommelier at a well-known restaurant in the city. His primary hobby is competitive bicycle racing, and he has a 150-km race coming up next weekend for which he has been training for the last 6 months. The race coordinators require every competitor to complete a comprehensive cardiovascular assessment prior to competing, and thus he comes to visit your clinic. On your assessment, you note a resting heart rate or 45 bpm with an occasional pause of up to 2 seconds. His blood pressure is 108/72 mmHg. He feels well and reports no syncopal or presyncopal episodes at rest or during exercise. Aside from the bradycardia, you note no other abnormalities on his examination. His ECG shows sinus rhythm with a PR interval of 128 msec, QRS duration of 80 msec, and occasional pauses as long as 2.2 seconds. Which of the following would be appropriate advice for Mr. Hendricks?
++
++
++
A. Perform electrophysiologic evaluation for consideration of a pacemaker.
++
++
B. He should not compete in the upcoming race due to concern for bradycardia and will need a tilt-table test.
++
++
C. No further follow-up is needed. Good luck with the race!
++
++
D. Obtain a 48-hour Holter monitor tracing.
++
++
E. He should undergo a treadmill stress ECG to determine the presence of chronotropic competence.
+
++
The answer is C. (Chap. 274) Sinus bradycardia and pauses up to 3 seconds are common in young individuals, particularly in highly trained athletes. Given the normal conduction intervals on his baseline ECG, lack of symptoms, and normal physical examination, no further testing is warranted. It is clear that he has the ability to augment his heart rate if he has been training for this race.
++
++
++
A 60-year-old man is undergoing an electrophysiology study for evaluation of syncope. After careful venous cannulation and placement of conductance and pacing catheters and after administration of 0.2 mg/kg of propranolol and 0.04 mg/kg of atropine, his heart rate is 65 bpm. After stopping the drugs and allowing adequate time for washout, his superior/lateral right atrium is paced 140 bpm. On cessation of this overdrive pacing, his next sinus beat occurs 1800 msec later. Based on these observations, this patient can be diagnosed with which of the following?
++
++
++
A. Amyloid cardiomyopathy
++
++
++
++
C. Paroxysmal atrial fibrillation
++
++
++
++
+
++
The answer is D. (Chap. 274) Determining the intrinsic heart rate (IHR) may distinguish SA node dysfunction from slow heart rates that result from high vagal tone. The normal IHR after administration of 0.2 mg/kg of propranolol and 0.04 mg/kg of atropine is 117.2 – (0.53 × age) in bpm; a low IHR is indicative of SA disease. For this patient, his IHR should be approximately 85 bpm. Electrophysiologic testing may play a role in the assessment of patients with presumed SA node dysfunction and in the evaluation of syncope, particularly in the setting of structural heart disease. In this circumstance, electrophysiologic testing is used to rule out more malignant etiologies of syncope, such as ventricular tachyarrhythmias and AV conduction block. There are several ways to assess SA node function invasively. They include the sinus node recovery time (SNRT), which is defined as the longest pause after cessation of overdrive pacing of the right atrium near the SA node (normal: <1500 msec). This patient has no evidence of tachyarrhythmias (option E) or atrial fibrillation (option B). While amyloid cardiomyopathy might explain SA nodal dysfunction, further evidence (endomyocardial biopsy or serologic evidence of light chain disease) would be required to make this diagnosis.
++
++
++
All of the following are reversible causes of SA node dysfunction EXCEPT:
++
++
++
++
++
++
++
C. Increased intracranial pressure
++
++
++
++
+
++
The answer is E. (Chap. 274) Sinoatrial dysfunction is often divided into intrinsic disease and extrinsic disease of the node. This is a critical distinction, because extrinsic causes are often reversible and pacemaker placement is not required. Drug toxicity is a common cause of extrinsic, reversible sinoatrial dysfunction, with common culprits including β-blockers, calcium channel blockers, lithium toxicity, narcotics, pentamidine, and clonidine. Hypothyroidism, sleep apnea, hypoxia, hypothermia, and increased intracranial pressure are all also reversible forms of extrinsic dysfunction. Radiation therapy can result in permanent dysfunction of the node and therefore is an irreversible, or intrinsic, cause of SA node dysfunction. In symptomatic patients, pacemaker insertion may be indicated.
++
++
++
Which of the following is a risk factor for development of thromboembolism in patients with the tachycardia-bradycardia variant of sick sinus syndrome?
++
++
++
++
++
++
++
++
++
D. Prothrombin 20210 mutation
++
++
E. None of the above; there is no increased risk of thromboembolism with the tachycardia-bradycardia variant of sick sinus syndrome
+
++
The answer is B. (Chap. 274) The tachycardia-bradycardia variant of sick sinus syndrome is associated with an increased risk of thromboembolism particularly when similar risk factors are present that increase the risk of thromboembolism in patients with atrial fibrillation. Specific risk factors associated with highest risk include age >65 years, patients with prior history of stroke, valvular heart disease, left ventricular dysfunction, or atrial enlargement. Patients with these risk factors should be treated with anticoagulation.
++
++
++
Normal cells within the AV node exhibit a property known as decremental conduction. If you wanted to demonstrate this property during an electrophysiology study, what maneuver could you perform?
++
++
++
A. Pace the right atrium at serially decreasing cycle length and measure the pace-to-His conduction time.
++
++
B. Pace the ventricle and record right atrial potentials.
++
++
C. Administer atropine 0.04 mg/kg and record AV nodal conduction time by measuring the time it takes an atrial pacing beat to reach the His bundle.
++
++
D. Administer metoprolol 10 mg intravenously and record AV nodal conduction time by measuring the time it takes an atrial pacing beat to reach the His bundle.
++
++
E. Administer adenosine 12 mg intravenously and record the AV nodal recovery time.
+
++
The answer is A. (Chap. 275) AV nodal transitional connections may exhibit decremental conduction, defined as slowing of conduction with increasingly rapid rates of stimulation. This property of the AV node is physiologically important. If a very rapid atrial arrhythmia occurs, the AV node acts as a sort of “gatekeeper” to the ventricles due to its decremental conduction. Thus, in the case of atrial fibrillation (where atrial rates often exceed 300 bpm), the ventricular rate never approaches these very rapid rates. Some accessory conduction pathways, in contrast, do not exhibit decremental conduction properties and will rapidly conduct tachycardias (such as atrial fibrillation) to the ventricles, leading to hemodynamic collapse.
++
++
++
An 87-year-old man with a history of well-treated hypertension and aortic stenosis has become symptomatic from his aortic stenosis over the last 2 months. Yesterday, he underwent surgical aortic valve replacement with a bioprosthetic 25-mm valve with excellent intraoperative results. He was rapidly weaned from cardiopulmonary bypass and extubated within 24 hours. Per surgical protocol, he had temporary epicardial pacing wires placed on the ventricular surface and has been pacing at 90 bpm. On rounds this morning, you briefly pause his ventricular pacing to check his underlying rhythm. You note an atrial rate of 80 bpm, but a ventricular rate of 32 bpm with a wide QRS complex. There is no relationship between the P waves and QRS complexes. This patient’s ventricular bradycardia is most likely due to which of the following?
++
++
++
A. Development of a systemic disease such as sarcoidosis or Lyme disease causing AV node dysfunction
++
++
B. Endocarditis of the new aortic bioprosthesis causing AV node block
++
++
++
++
D. Slowed AV nodal recovery after overdrive pacing
++
++
E. Surgical injury to the AV node
+
++
The answer is E. (Chap. 275) The compact AV node (~1 × 3 × 5 mm) is situated at the apex of the triangle of Koch, which is defined by the coronary sinus ostium posteriorly, the septal tricuspid valve annulus anteriorly, and the tendon of Todaro superiorly. The compact AV node continues as the penetrating AV bundle where it immediately traverses the central fibrous body and is in close proximity to the aortic, mitral, and tricuspid valve annuli; thus, it is subject to injury in the setting of valvular heart disease or its surgical treatment. It is common for patients to experience transient AV block after valve surgery (particularly aortic valve surgery) due to the surrounding edema. Many patients will regain normal conduction as the perioperative injury and edema decrease; however, some patients will not and will require permanent pacemaker placement. It is unlikely that this patient has developed new systemic disease such as Lyme disease, sarcoidosis, or endocarditis. SA nodal disease would manifest as sinus bradycardia, which this patient does not have.
++
++
++
Mrs. Hellwig is a 25-year-old woman with systemic lupus erythematosus (SLE) complicated by nephropathy, hemolytic anemia, and pleuritis. Her disease is well controlled on therapy. She recently discovered that she is pregnant, and presents today for prenatal counseling. She specifically is concerned about the effect her autoimmune disease may have on the infant. You tell her that the most common cardiac complication in children born to mothers with SLE is:
++
++
++
++
++
B. Coronary artery disease
++
++
C. Dilated cardiomyopathy
++
++
D. Pulmonary hypertension with right ventricular (RV) failure
++
++
E. Sterile, Libman-Sacks endocarditis
+
++
The answer is A. (Chap. 275) Congenital AV block in the setting of a structurally normal heart has been seen in children born to mothers with systemic lupus erythematosus (SLE). SLE can cause sterile endocarditis in adults but is not common in children of mothers with SLE. Similarly, early-onset coronary artery disease, pulmonary hypertension, and occasionally cardiomyopathy are described in SLE patients but not in their children just after birth.
++
++
++
Mr. Hoffman, an 82-year-old former tightrope performer, presents to your office for complaints of syncope. He states that twice in the past week, he has spontaneously passed out with no warning symptoms. Once, he struck his face, and you note that he has periorbital ecchymosis on exam. Other than this, you find nothing abnormal on examination. You request an ECG and step out of the room to begin your documentation. Shortly thereafter, your medical assistant requests your urgent presence in Mr. Hoffman’s clinic room. He had another “spell” during the ECG and lost consciousness. Serendipitously, the medical assistant captured the spell on ECG, pictured in Figure V-30. What type of AV block is present and is matched to the appropriate treatment or diagnostic test?
+
+
++
+
++
++
++
A. Complete SA nodal block – Permanent pacemaker implantation
++
++
B. First-degree AV block – Administer atropine
++
++
C. Second-degree Mobitz type II SA nodal block – Exercise treadmill study
++
++
D. Second-degree Mobitz type I AV block: –No intervention necessary
++
++
E. Second-degree Mobitz type II AV node block – Permanent pacemaker implantation
+
++
The answer is E. (Chap. 275) This patient has Mobitz II, second-degree AV block. This particular ECG demonstrates a specific pattern of this called paroxysmal AV block. This patten and the patient’s concerning symptoms of recurrent syncope indicate significant conduction disease and warrant urgent permanent pacemaker implantation. SA node block would be indicated on the surface ECG by the absence of P waves (and SA node potentials might be noted on invasive intracardiac electrograms). First-degree AV block would manifest as a prolonged PR interval (>200 msec) without any nonconducted beats. Atropine (anticholinergic) administration can sometimes help differential between Mobitz I (usually intranodal disease) and Mobitz II (usually infranodal disease) block when the ECG is not clear. Since the AV node is more heavily innervated by vagal efferents, atropine will improve the block in Mobitz I but worsen it in Mobitz II infranodal disease. The converse is largely true of maneuvers that increase vagal tone (carotid sinus massage or Valsalva).
++
++
++
A 19-year-old long-distance runner, who finished in the top 10 of the local marathon last year, presents for cardiac evaluation after his primary care physician ordered a Holter monitor for screening purposes. On his Holter report, several episodes of second-degree, Mobitz I (Wenckebach) AV block were noted, all occurring during sleep. The patient reports no symptoms but thinks he may have a grandfather who had a pacemaker implanted at an advanced age. What is the most appropriate next step?
++
++
++
A. Exercise treadmill stress ECG
++
++
B. Invasive electrophysiology study
++
++
++
++
D. Refer for pacemaker implantation
++
++
E. Serologic testing including thyroid-stimulating hormone levels
+
++
The answer is C. (Chap. 275) Mobitz I (Wenckebach) AV block is a very common phenomenon in young, healthy adults, particularly during sleep and in patients with high vagal tone such as trained athletes. Given his reassuring lack of symptoms, no further testing or intervention is needed for this patient. The indications for pacemaker implantation in AV block are noted in Table V-31. Note that asymptomatic type I second-degree AV block is a class III recommendation (recommended against).
++
++
++
++
A 47-year-old woman with a history of tobacco abuse and ulcerative colitis is evaluated for intermittent palpitations. She reports that for the last 6 months, every 2–4 days she notes a sensation of her heart “flip-flopping” in her chest for approximately 5 minutes. She has not noted any precipitating factors and has not felt light headed or had chest pains with these episodes. Her physical examination is normal. A resting ECG reveals sinus rhythm and no abnormalities. Aside from checking serum electrolytes, which of the following is the most appropriate testing?
++
++
++
A. Abdominal computed tomography (CT) with oral and intravenous (IV) contrast
++
++
++
++
++
++
D. Reassurance with no further testing needed
++
++
E. Referral for electrophysiology study
+
++
The answer is B. (Chap. 276) The patient has persistent, non–life-threatening palpitations that distress her enough to seek medical attention. A continuous Holter monitor for 24 hours is appropriate for patients in whom the symptoms happen several times over 24 hours, whereas an event monitor is triggered by the patient when symptoms occur and thus can be worn for longer period of time, which is appropriate in this patient. There is no indication of gastrointestinal triggers, so abdominal computed tomography (CT) would not be helpful. The atrial premature contractions are uncomplicated, do not require additional diagnostic evaluation at this time, and pose no additional health risk. Electrophysiology (EP) referral is indicated for patients with life-threatening or severe symptoms such as syncope.
++
++
++
A 47-year-old woman with a history of tobacco abuse and ulcerative colitis is evaluated for intermittent palpitations. She reports that for the last 6 months, every 2–4 days she notes a sensation of her heart “flip-flopping” in her chest for approximately 5 minutes. She has not noted any precipitating factors and has not felt light headed or had chest pains with these episodes. Her physical examination is normal. A resting ECG reveals sinus rhythm and no abnormalities. After further testing, the patient is found to have several episodes of atrial premature contractions. Which of the following statements regarding the dysrhythmia in this patient is true?
++
++
++
A. Atrial premature contractions are less common that ventricular premature contractions on extended ECG monitoring.
++
++
B. Echocardiography is indicated to determine if structural heart disease is present.
++
++
C. Metoprolol should be initiated for symptom control.
++
++
D. The patient should be reassured that this is not a dangerous condition and does not require further evaluation.
++
++
E. The patient should undergo stress test to determine if ischemia is present.
+
++
The answer is D. (Chap. 276) The patient has persistent, non–life-threatening palpitations that distress her enough to seek medical attention. A continuous Holter monitor for 24 hours is appropriate for patients in whom the symptoms happen several times over 24 hours, whereas an event monitor is triggered by the patient when symptoms occur and thus can be worn for longer period of time, which is appropriate in this patient. There is no indication of gastrointestinal triggers, so abdominal computed tomography (CT) would not be helpful. The atrial premature contractions are uncomplicated, do not require additional diagnostic evaluation at this time, and pose no additional health risk. Electrophysiology (EP) referral is indicated for patients with life-threatening or severe symptoms such as syncope.
++
++
++
Ms. Milsap is an 18-year-old high school volleyball star with a sports scholarship to the local university. As part of her admission process, she is required to undergo a full medical assessment prior to taking part in collegiate sports. Physical examination reveals no abnormalities, although she reports the rare episode of palpitations and light-headedness. ECG reveals a PR interval of 0.06 msec, QRS duration of 140 msec, and a slurred upstroke or delta wave in the initial part of the QRS. You correctly diagnose this as Wolff-Parkinson-White pattern. Which of the following findings is reassuring that Ms. Milsap will suffer no ill effects or need catheter ablation due to this abnormality?
++
++
++
A. Ability to increase the heart rate to 185 bpm on an exercise treadmill test
++
++
B. Electrophysiology study demonstrating that the accessory pathway has both antegrade and retrograde conduction properties
++
++
C. Electrophysiology study demonstrating that the accessory pathway is located in the posteroseptal region.
++
++
D. Exercise treadmill study demonstrating disappearance of the delta wave and wide QRS at a heart rate of 120 bpm
++
++
E. Holter monitoring demonstrating occasional runs of atrial fibrillation
+
++
The answer is D. (Chap. 276) Wolff-Parkinson-White (WPW) pattern is almost always due to accessory pathway conduction whereby electrical signals are able to propagate from atria to ventricles without first going through the AV node. As opposed to the AV node, many accessory pathways fail to exhibit decremental conduction (a slowing of conduction at increasing rates of excitation) and thus are able to rapidly conduct fast atrial rhythms to the ventricles. At exceedingly fast rates (as might be present in atrial fibrillation), this can lead to cardiovascular collapse. The observation that Ms. Milsap’s delta wave disappears and QRS complex normalizes at a relatively high heart rate is reassuring. In that case, the accessory pathway cannot conduct antegrade at a rate >120 bpm and is unlikely to cause serious tachyarrhythmia. The ability to augment a rapid sinus rhythm is normal for her age and does not carry any particular prognosis. The location of the accessory pathway in a septal position makes is it somewhat more difficult to ablate with catheters, and the operator must take care to avoid the native AV node and His-Purkinje system. The ability of the accessory pathway to conduct both antegrade and retrograde makes it a substrate for atrioventricular reentrant tachycardia (AVRT).
++
++
++
An 85-year-old woman with no prior cardiac history presents to the emergency department with 2 hours of palpitations. Blood pressure, oxygen saturation, and heart rate are normal, although you note an irregularly irregular rhythm on examination. ECG shows an irregularly irregular, narrow QRS without discernable P waves at a rate of 75 bpm. Echocardiogram reveals no structural heart disease. Despite the normal heart rate, the patient is quite symptomatic with her atrial fibrillation and wants to pursue achievement of sinus rhythm. All of the following interventions may be beneficial EXCEPT:
++
++
++
A. Adenosine intravenously
++
++
B. Amiodarone intravenously
++
++
C. Direct current cardioversion
++
++
++
++
+
++
The answer is A. (Chap. 276) Adenosine is a powerful “nodal-blocking agent”; that is, it blocks propagation of electrical signal through the AV node. It has a very short half-life. Administration of adenosine to a patient in atrial fibrillation will induce a transient complete heart block but will not stop the atrial fibrillation. When the adenosine wears off in a few seconds and AV conduction resumes, the ventricular rhythm will again be irregularly irregular. All the other choices have been shown to achieve sinus rhythm better than placebo in patients with atrial fibrillation.
++
++
++
A 79-year-old man with a history of coronary artery disease, ischemic cardiomyopathy with a last left ventricular (LV) ejection fraction of 30%, and hypertension presents to your office with no new complaints. Blood pressure is 108/56 mmHg, heart rate is 88 bpm, and arterial oxygen saturation is 98%. His rhythm strip is shown in Figure V-36. Based on this ECG, the patient now has a definite (class I) indication for which of the following therapies?
+
+
++
+
++
++
++
A. Amiodarone 400 mg daily
++
++
++
++
C. Flecainide 600 mg PRN palpitations
++
++
D. Systemic anticoagulation with warfarin or a novel oral anticoagulant
++
++
E. Transesophageal echocardiography followed by direct current (DC) cardioversion
+
++
The answer is D. (Chap. 276) This patient has new atrial fibrillation. When one assesses a patient with new atrial fibrillation, it is prudent to proceed through a series of decisions systematically. If the patient is hemodynamically unstable (low blood pressure, pulmonary edema, poor mentation, low urine output), then urgent direct current cardioversion is warranted. This patient is clearly stable. The next decision hinges around rate versus rhythm control. Several studies have shown clinical equipoise between rate and rhythm control strategies, with patients randomized to the rhythm control strategies undergoing far more procedures and taking more medications than patients randomized to the rate control strategies. This is also true in heart failure patients. Given the lack of symptoms and already controlled resting heart rate, rhythm control with either medications (amiodarone) or cardioversion is not a class I indication. In fact, a type I antiarrhythmic such as flecainide would be contraindicated for this patient because it has been shown to increase mortality in patients with coronary disease. The final question revolves around anticoagulation. Patients with atrial fibrillation have varying degrees of risk of thromboembolic events. Patients are stratified into risk categories by assessing set risk factors. One can remember the CHADS2 mnemonic to recall each point of the risk scoring system (congestive heart failure [CHF], hypertension, age >75, diabetes, and stroke/cerebrovascular accident [which receives 2 points]). A more sensitive score includes vascular disease, age >65, and female sex (CHADS-VASc). Any combined score >1 warrants systemic anticoagulation. Patients with a score of 0 do not require systemic anticoagulation and can take full-strength aspirin. A score of 1 is intermediate and requires an in-depth discussion with the patient about their risk threshold for anticoagulation. Many experts recommend systemic anticoagulation with a CHADS2 score of 1. This patient has a CHADS2 score of 3 (CHF, hypertension, age) and thus warrants systemic anticoagulation.
++
++
++
A 43-year-old woman is seen in the emergency department after sudden onset of palpitations 30 minutes prior to her visit. She was seated at her work computer when the symptoms began. Aside from low back pain, she is otherwise healthy. In triage, her heart rate is 178 bpm and blood pressure is 98/56 mmHg with normal oxygen saturation. On physical examination, she has a “frog sign” in her neck and tachycardia but is otherwise normal. ECG shows a narrow complex tachycardia without identifiable P waves. Which of the following is the most appropriate first step to manage her tachycardia?
++
++
++
++
++
++
++
++
++
++
++
E. DC cardioversion using 100 J
+
++
The answer is D. (Chap. 276) This patient has classic symptoms for an AV nodal reentrant tachycardia. The so-called frog sign (prominent venous pulsations in the neck due to cannon A waves seen in AV dissociation) on physical examination is frequently present and suggests simultaneous atrial and ventricular contraction. First-line therapy for these reentrant narrow complex tachyarrhythmias is carotid sinus massage to increase vagal tone. Often this is all that is required to return the patient to sinus rhythm. If that is not successful, intravenous (IV) adenosine 6–12 mg may be attempted. If adenosine fails, IV β-blockers or calcium channel blockers may be used (diltiazem or verapamil). Finally, in hemodynamically compromised patients or those who have failed to respond to previous measures, direct current (DC) cardioversion with 100–200 J is indicated.
++
++
++
Which of the following statements regarding restoration of sinus rhythm after atrial fibrillation is true?
++
++
++
A. Dofetilide may be safely started as an outpatient.
++
++
B. In patients who are treated with pharmacotherapy and are found to be in sinus rhythm, a prolonged Holter monitor should be worn to determine if anticoagulation can be safely stopped.
++
++
C. Patients who have pharmacologically maintained sinus rhythm after atrial fibrillation have improved survival compared with patients who are treated with rate control and anticoagulation.
++
++
D. Recurrence of atrial fibrillation is uncommon when pharmacotherapy is used to maintain sinus rhythm.
+
++
The answer is B. (Chap. 276) The AFFIRM and RACE trials compared outcomes in survival and thromboembolic events in patients with atrial fibrillation using two treatment strategies: rate control and anticoagulation versus pharmacotherapy to maintain sinus rhythm. There was no difference in events in the two groups, which was thought to be due to the inefficiencies of pharmacotherapy, with over half of patients failing drug therapy, and also the high rates of asymptomatic atrial fibrillation in the sinus rhythm group. Thus, when considering discontinuation of anticoagulation in patients who have maintained sinus rhythm, it is recommended to place a prolonged ECG monitor to ensure that asymptomatic atrial fibrillation is not present. Because of the risk of QT prolongation and polymorphic ventricular tachycardia, dofetilide and sotalol are recommended to be initiated in the hospital.
++
++
++
A 76-year-old woman with a history of hypertension, chronic obstructive pulmonary disease (COPD), diabetes mellitus, and osteoporosis presents to the emergency department after a fall at home followed immediately by intense left hip pain. She was found by a neighbor after several hours. The patient cannot remember if she lost consciousness. She is exquisitely tender to palpation over the left hip, and her leg is shortened and externally rotated. Her mucous membranes are dry, and her skin tents easily. Blood pressure is 170/80 mmHg and heart rate is 130 bpm. Her rhythm strip is shown in Figure V-39. What is the most appropriate first step for her tachycardia?
+
+
++
+
++
++
++
++
++
++
++
++
++
++
++
E. Pain control and IV hydration
+
++
The answer is E. (Chap. 276) This rhythm is multifocal atrial tachycardia (MAT), which is easily confused with atrial fibrillation due to its irregularly irregular nature. However, MAT is distinguished by the multiple different P waves present (at least three different P-wave morphologies). MAT is classically present in patients with severe pulmonary disease and is exacerbated in the presence of acute illness. No anticoagulation is needed for MAT, and DC cardioversion is not efficacious. Some patients respond to the nondihydropyridine calcium channel blockers (diltiazem and verapamil), and amiodarone has some limited effect. In this patient, controlling the underlying cause of her tachycardia (pain and dehydration) is the most appropriate first therapy.
++
++
++
A young woman is brought to the emergency department after witnesses observed her suddenly lose consciousness while jogging in the nearby park. She has a fractured nose and broken teeth from her fall. She does not have identification. She is nonresponsive to painful stimuli. Blood pressure is 50/palp and heart rate is close to 280 bpm. A 12-lead ECG is shown in Figure V-40. What is the next most appropriate step?
+
+
++
+
++
++
++
A. Amiodarone 150 mg IV stat
++
++
B. Lidocaine 1 mg IV stat
++
++
++
++
++
++
E. Emergent CT scan with contrast to evaluate for pulmonary embolism
+
++
The answer is D. (Chap. 276) This patient is unstable with hypotension and poor cerebral perfusion due to her wide complex tachycardia. Defibrillation is the only therapy that is appropriate and should be performed immediately.
++
++
++
A young woman is brought to the emergency department after witnesses observed her suddenly lose consciousness while jogging in the nearby park. She has a fractured nose and broken teeth from her fall. She does not have identification. She is nonresponsive to painful stimuli. Blood pressure is 50/palp and heart rate is close to 280 bpm. A 12-lead ECG is shown in Figure V-40. What is the underlying rhythm in the patient?
+
+
++
+
++
++
++
A. Atrial fibrillation with antegrade conduction through an accessory tract
++
++
B. Atrial flutter with 1:1 conduction
++
++
C. AV reentry tachycardia
++
++
D. AV nodal reentry tachycardia
++
++
E. Ventricular tachycardia
+
++
The answer is A. (Chap. 276) This ECG shows a wide complex, irregularly irregular tachycardia. Ventricular tachycardia is wide complex but is regular. Unless there is underlying conduction system disease (such as a left bundle branch block, which is unusual in a young patient), AV nodal reentry tachycardia (AVNRT), AVRT, and atrial flutter are also narrow complex and are regular. The irregularly irregular rhythm coupled with the extremely fast ventricular rate make the diagnosis of atrial fibrillation with antegrade conduction down an accessory pathway most likely. If no accessory pathway is present, the decremental conduction properties of the AV node will limit ventricular response rate. However, some accessory pathways do not have decremental conduction properties, and thus pass every atrial impulse through to the ventricle. In the setting of atrial rates above 300 (as in atrial fibrillation), this can be catastrophic.
++
++
++
A 67-year-old man with a history of hypertension and hyperlipidemia presented 3 hours ago to the emergency department with crushing substernal chest pain and dyspnea of acute onset. There, ST elevations were noted in the anterior and lateral leads on his ECG, and thrombolytics were administered. He was admitted to the cardiac intensive care unit. Now, his nurse calls you to his room because he is having unusual tracings on his monitor. A sample rhythm strip recorded from lead V6 is shown in Figure V-42. What rhythm does this represent?
+
+
++
+
++
++
++
++
++
B. Complete heart block with a junctional escape
++
++
C. Idioventricular rhythm
++
++
D. Normal sinus rhythm with a left bundle branch block
++
++
E. Ventricular tachycardia
+
++
The answer is C. (Chap. 277) This is idioventricular rhythm, which is a rhythm originating in the ventricular myocardium but with a rate <100 bpm. This rhythm, also known as accelerated idioventricular rhythm, is commonly seen after successful reperfusion and requires no specific therapy. Normal sinus rhythm is ruled out by the absence of P waves. Ventricular tachycardia requires a heart rate of >100 bpm for diagnosis. A junctional escape would likely be narrow (unless there is aberrant conduction) and much slower (40–60 bpm). Atrial fibrillation would be irregularly irregular.
++
++
++
You are having a quiet shift in the local emergency department. For the past 4 days, there has been a large snowstorm limiting traffic, and people have generally stayed indoors and out of harm’s way. Suddenly, the emergency medical personnel arrive with a middle-aged woman who is obtunded. Her downstairs neighbors noted that they had not seen her for the past 4 days (since the snow started) and alerted the police who found her unconscious on her floor. On your assessment, you can barely feel a radial pulse. You note a dialysis fistula graft in her left arm. Blood pressure is 60/palp, and her ECG tracing is shown in Figure V-43. You rapidly set up for defibrillation and request a full chemistry panel. What electrolyte abnormality do you expect to find?
+
+
++
+
++
++
++
++
++
++
++
++
++
++
++
+
++
The answer is B. (Chap. 277) This ECG shows a slow, sinusoidal ventricular rhythm. This bizarre rhythm is almost always due to drug effects or electrolyte disturbances. Specifically, this represents the most extreme ECG changes of hyperkalemia. Unfortunately, the scenario in this question is all too common in patients on hemodialysis who cannot or do not go to dialysis sessions. Hyperkalemia is a dreaded complication of missed dialysis and can be deadly. Hyperphosphatemia and resultant hypocalcemia can also be caused by missed hemodialysis sessions. These do not cause the observed ECG changes. Although defibrillation is appropriate, one should realize that it may not be effective until the hyperkalemia is controlled. Temporizing measures such as IV calcium, insulin and dextrose, bicarbonate, and inhaled β-agonists can be used to lower the serum potassium while emergent hemodialysis is readied.
++
++
++
A 71-year-old woman with ischemic cardiomyopathy and a left ventricular ejection fraction of 38% has been hospitalized for the past week with acute decompensated heart failure. After diuresis and medication optimization, she is feeling immensely better. She is on the maximally tolerated doses of an angiotensin-converting enzyme (ACE) inhibitor, β-blocker, and appropriate diuretic dose. You are planning for discharge today. On rounds, the nurse notes that the patient had several short (5–10 beats) runs of nonsustained ventricular tachycardia (NSVT) and multiple premature ventricular contractions (PVCs) overnight, although she remained asymptomatic. A medical student on the team asks if the NSVT carries any prognostic significance and if any intervention is needed. What is the most appropriate response?
++
++
++
A. “NSVT is common is patients with cardiomyopathy and carries no significance.”
++
++
B. “NSVT is concerning in this patient. We should suppress the PVCs and NSVT with amiodarone.”
++
++
C. “NSVT is associated with an increased mortality in patients with heart failure. We will refer her for an automated implantable defibrillator.”
++
++
D. “NSVT is associated with an increased mortality in patients with heart failure. We should suppress the NSVT with flecainide.”
++
++
E. “NSVT is associated with an increased mortality in patients with heart failure. However, suppression of PVCs and NSVT with antiarrhythmic drugs does not change this prognosis.”
+
++
The answer is E. (Chap. 277) Although it is true that patients with cardiomyopathy and nonsustained ventricular tachycardia (NSVT) or a high burden of premature ventricular contractions (PVCs) have a higher mortality, multiple studies have found no mortality benefit to suppressing the ventricular arrhythmias with antiarrhythmic medications. In fact, the CAST trial showed that the use of Vaughn-Williams class I drugs such as flecainide and propafenone increased mortality when used in patients with PVCs and ischemic disease. Thus, these are contraindicated in patients with any structural heart disease. Implantable defibrillator therapy is not warranted in this patient for primary prevention despite the presence of NSVT. It would be prudent to ensure that her electrolytes are normal given her recent diuresis; hypokalemia in particular is associated with an increased burden of PVCs and NSVT.
++
++
++
You are taking care of Mr. Wittstine in the cardiac intensive care unit. He is a 62-year-old man with hypertension, hyperlipidemia, and tobacco abuse who suffered a massive anterior myocardial infarction 4 days prior. He presented late to the emergency department, and thus despite expeditious primary coronary intervention with angioplasty and stent placement to his left anterior descending artery, echocardiogram today shows an ejection fraction of 25%. He has had no ventricular arrhythmias. He still feels slightly short of breath, and your examination reveals bilateral pulmonary rales and an elevated jugular venous pulse. Other than aspirin, clopidogrel, and atorvastatin, he is on no other therapy. All of the following therapies will reduce his mortality over the next 40 days EXCEPT:
++
++
++
A. Automated implantable defibrillator
++
++
++
++
++
++
++
++
E. All of the above will reduce Mr. Wittstine’s mortality
+
++
The answer is A. (Chap. 277) For survivors of an acute MI, an implantable cardioverter defibrillator (ICD) reduces mortality in certain high-risk groups: patients who have survived >40 days after the acute MI and have a left ventricular (LV) ejection fraction of ≤0.30 or who have an ejection fraction <0.35 and have symptomatic heart failure (functional class II or III). Thus, early ICD implantation is not warranted here. In patients >5 days after MI with NSVT and inducible sustained ventricular tachycardia (VT) or ventricular fibrillation (VF) on electrophysiologic studies, an automatic ICD may be considered. Angiotensin-converting enzyme (ACE) inhibitors, β-blockers, and eplerenone (aldosterone antagonists) have all been shown to reduce early mortality in patients after an anterior MI with reduced ejection fraction.
++
++
++
A 21-year-old college student who is studying musical education at the local university recently joined a chorus group. However, whenever she stands to sing a solo, she notes the onset of palpitations and light-headedness. While she has never lost consciousness, she occasionally has to sit down due to the dizziness. She presents to your clinic for evaluation. Her physical examination is completely normal, as is her baseline resting ECG. You ask her to sing a solo for the clinic staff and simultaneously record a 12-lead ECG (seen in Figure V-46). Her singing voice is lovely, but she quickly experiences the symptoms and has to stop. You refer her for echocardiography and cardiac MRI, which are both normal. You suspect that the patient’s palpitations are due to which of the following?
+
+
++
+
++
++
++
A. Arrhythmogenic right ventricular dysplasia
++
++
++
++
C. Hypertrophic cardiomyopathy
++
++
D. Left ventricular intrafascicular ventricular tachycardia
++
++
E. Right ventricular outflow tract ventricular tachycardia
+
++
The answer is E. (Chap. 277) This patient has a subset of idiopathic VT, or VT originating in the absence of structural or inheritable cardiac disease. The presence of a normal baseline ECG, echocardiogram, and cardiac magnetic resonance imaging (MRI) essentially rule out Brugada syndrome, arrhythmogenic right ventricular dysplasia, and hypertrophic cardiomyopathy (although these are all important considerations in the patients with ventricular arrhythmias at a young age). The pictured ECG is classic for right ventricular outflow tract (RVOT) VT with a left bundle branch block pattern (indicating origin in the RV) and inferior axis (indicating origin in the cranial part, or base, of the heart). LV intrafascicular VT is another idiopathic VT, but presents with a right bundle branch block pattern. RVOT VT is often induced by scenarios with sympathetic nervous system activation, such as singing a solo in front of a crowd. This arrhythmia is not associated with sudden cardiac death and often responds to β-blockers or nondihydropyridine calcium channel blockers. Catheter ablation is an option if medical therapy fails to control the symptoms or is not desired.
++
++
++
A 47-year-old woman who is maintained on methadone for a prior history of narcotic abuse recently contracted an upper respiratory tract infection, and her friend offered her some leftover erythromycin pills. Today, she felt multiple episodes of palpitations and light-headedness, prompting presentation to the emergency department. There, rhythm strip demonstrated multiple nonsustained runs of the arrhythmia featured in Figure V-47. You administer 2 mg of IV magnesium sulfate; however, the episodes of nonsustained arrhythmia do not abate. Her laboratory examination shows a normal potassium level. What is the next most appropriate step?
+
+
++
+
++
++
++
A. Metoprolol 5 mg IV repeatedly until her resting heart rate is 60 bpm
++
++
++
++
C. Emergent referral for implantable defibrillator
++
++
D. IV isoproterenol infusion titrated to a rate of 100–120 bpm
++
++
E. Sedation and defibrillation
+
++
The answer is D. (Chap. 277) This patients has medication-induced long QT syndrome (methadone and erythromycin are common causes) and resulting torsades de pointes (TdP). Sustained TdP is never a perfusing rhythm and can rapidly degenerate into ventricular fibrillation. For nonsustained TdP, correction of hypokalemia and expeditious administration of IV magnesium may lead to cessation of the arrhythmia. If this fails, administration of isoproterenol or pacing to a rate of 100–120 bpm will suppress PVCs and lead to rate-dependent shortening of the QT interval, thereby suppressing TdP and allowing time for the medications to washout. Inducing bradycardia (with metoprolol) or further QT prolongation with amiodarone may be harmful and lead to more TdP. Defibrillation is not warranted in NSVT. Likewise, ICD implantation is not necessary given that this is a secondary phenomenon due to drug interaction. This patient should be very cautious to avoid QT-prolonging agents in the future, however, and should consider transitioning off methadone at the earliest chance.
++
++
++
In an ECG with wide complex tachycardia, which of the following clues most strongly supports the diagnosis of ventricular tachycardia?
++
++
++
A. Atrial-ventricular dissociation
++
++
B. Classic right bundle branch block pattern
++
++
C. Irregularly irregular rhythm with changing QRS complexes
++
++
D. QRS duration >120 msec
++
++
E. Slowing of rate with carotid sinus massage
+
++
The answer is A. (Chap. 277) Atrial-ventricular dissociation is a classic finding in ventricular tachycardia. Physical examination may show jugular vein cannon a waves when the atria contracts against a closed tricuspid valve, and the ECG will manifest this with atrial capture and/or fusion beats. Other findings on ECG of ventricular tachycardia include QRS duration >140 msec for right bundle branch pattern in V1 or >160 msec for left bundle morphology in lead V1, frontal plane axis –90 to 180 degrees, delayed activation during initial phase of the QRS complex, and bizarre QRS pattern that does not mimic typical right or left bundle branch block QRS complex patterns. An irregularly irregular rhythm with changing QRS complexes suggests atrial fibrillation with ventricular preexcitation. Carotid sinus massage, aimed at increasing vagal tone and slowing AV node conduction, is not effective at slowing ventricular tachycardia because the reentrant focus is below the AV node.
++
++
++
An 18-year-old man with no prior past medical history presents for his required medical assessment before beginning his freshman year at the local university. His history and examination uncover no concerning symptoms or signs. However, his ECG shows an irregular rhythm and is pictured in Figure V-49. What is the most appropriate next step?
+
+
++
+
++
++
++
++
++
B. Electrophysiology study and planned ablation of an ectopic atrial focus
++
++
C. Exercise ECG stress test
++
++
D. Holter monitor to define arrhythmia burden
++
++
+
++
The answer is E. (Chap. 278e) This ECG demonstrates sinus arrhythmia, a normal physiologic finding particularly in young adults. During inspiration, the sinus rate speeds up to maintain cardiac output in the face of a slightly decreased left ventricular stroke volume, and vice versa during expiration. This is a normal, healthy heart rhythm and requires nor further investigation or intervention.
++
++
++
Ms. Hardy is a 22-year-old triathlete with a history of type 1 diabetes mellitus. Unfortunately, her pharmacy provided her with an expired batch of insulin due to a clerical error, and she presented to the hospital in diabetic ketoacidotic crisis 3 days ago. With aggressive hydration, electrolyte repletion, and insulin administration, her ketoacidotic crisis was reversed. She is feeling much better and is planned for discharge tomorrow. You are called by the monitor station and told that Ms. Hardy is bradycardic. Her ECG is shown in Figure V-50. You check on her and note that she is sleeping soundly. The anatomic location of her heart block is likely where?
+
+
++
+
++
++
++
++
++
++
++
++
++
++
++
+
++
The answer is A. (Chap. 278e) This ECG shows 2:1 second-degree AV block; that is, there are two P waves for every QRS complex. In this type of second-degree AV block, it can be difficult to tell the difference between type I (Wenckebach) block, which is usually anatomically located at the AV node, and type II block, which is often infranodal in origin. There is no progressively prolonging or stable PR interval to distinguish. However, the presence of a narrow, normal QRS complex is sufficient to make Wenckebach block most likely, particularly in an athletic young person during sleep, a time of high vagal tone. A wide QRS complex during 2:1 conduction, particularly in a patient in whom infranodal disease would be likely, is concerning for second-degree type II block and more concerning for progression to complete or high-grade heart block.
++
++
++
Ms. Hardy is a 22-year-old triathlete with a history of type 1 diabetes mellitus. Unfortunately, her pharmacy provided her with an expired batch of insulin due to a clerical error, and she presented to the hospital in diabetic ketoacidotic crisis 3 days ago. With aggressive hydration, electrolyte repletion, and insulin administration, her ketoacidotic crisis was reversed. She is feeling much better and is planned for discharge tomorrow. You are called by the monitor station and told that Ms. Hardy is bradycardic. Her ECG is shown in Figure V-50. You check on her and note that she is sleeping soundly. If you were to administer atropine intravenously to Ms. Hardy, the patient, you would expect which of the following to occur?
+
+
++
+
++
++
++
A. Increase sinus rate and maintain 2:1 AV block
++
++
B. Increase sinus rate and improve heart block to 1:1 conduction
++
++
C. Increase sinus rate and worsen heart block to >2:1
++
++
D. No change in sinus rate and worsen heart block to >2:1
++
++
+
++
The answer is B. (Chap. 278e) In heart block occurring at the AV node, the administration of atropine (an anticholinergic agent) will often improve the block as the AV node has a high degree of vagal efferent innervation. The SA node phase 4 action potential slope will increase as well with anticholinergic stimulation, causing faster sinus rate. In heart block occurring below the AV node, anticholinergic stimulation will often cause the appearance of a worse (3:1, 4:1, etc.) heart block as the sinus rate increases and more impulses are transmitted to the infranodal tissue during its prolonged (and unchanged with atropine) refractory period. One may use this finding to differentiate Wenckebach and type II second-degree heart block during difficult-to-interpret 2:1 conduction.
++
++
++
An 87-year-old man with Parkinson disease takes levodopa. After running out of his medication and failing to obtain a refill, he presents to the emergency department with a parkinsonian exacerbation. On initial evaluation, he has the ECG shown in Figure V-52. Which of the following is the next most appropriate step?
+
+
++
+
++
++
++
A. Anticoagulation with IV heparin
++
++
B. Check serum thyroid-stimulating hormone (TSH) and obtain targeted respiratory history
++
++
++
++
++
++
E. No further testing or intervention
+
++
The answer is E. (Chap. 278e) This ECG illustrates tremor artifact, most notable in leads I, III, and aVL (thus making it likely that the tremor is occurring in the left arm). It is easy to confuse tremor artifact with atrial flutter or atrial fibrillation. Note the normal sinus P waves in lead II and all the precordial leads. No further testing is needed for this ECG.
++
++
++
Mr. Tillman is a 68-year-old man with a history of nonischemic cardiomyopathy, hypertension, and a transient ischemic attach 6 months ago with no obvious cause after appropriate workup. He presents to the emergency department with 5 days of not feeling well. On examination, he appears tired. Blood pressure is 110/78 mmHg, and his heart rate is 150 bpm. His jugular veins are not distended and lungs are clear. His ECG is pictured in Figure V-53. You appropriately administer 5 mg of IV metoprolol twice, which brings his heart rate down to 75 bpm. He feels much better. You consult the cardiac electrophysiologist for consideration of ablation of the etiologic arrhythmia. What is the next most appropriate step?
+
+
++
+
++
++
++
++
++
++
++
++
++
++
++
E. Systemic anticoagulation
+
++
The answer is E. (Chap. 278e) This ECG shows atrial flutter, with 2:1 conduction and the typical atrial rate of 300 bpm and ventricular rate of 150 bpm. The saw-tooth baseline pattern, best appreciated in the inferior leads, is a clue that this atrial flutter is “typical,” or due to a macro-reentrant circuit that relies on the cavo-tricuspid isthmus. As such, it is quite amenable to ablation with a >95% success rate. However, given that Mr. Tillman’s symptoms have been ongoing for >48 hours, he is at risk for development of an atrial thrombus (similar to in atrial fibrillation) and thus requires systemic anticoagulation before achievement of sinus rhythm. He will also likely require a transesophageal echocardiogram to ensure that no clot exists prior to ablation or cardioversion. Aspirin is not adequate to prevent potential embolization. Atrial flutter will generally not convert in response to amiodarone, digoxin, or other medications. Given the absence of heart failure, there is no indication for furosemide.
++
++
++
Ms. Schoop is a 22-year-old pottery store owner who presents to you for evaluation of palpitations. Her ECG is pictured in Figure V-54. She likely has a bypass accessory tract located in the:
+
+
++
+
++
++
++
++
++
B. Anterolateral left ventricle
++
++
C. Inferior left ventricle
++
++
++
++
+
++
The answer is E. (Chap. 278e) This ECG is diagnostic of Wolff-Parkinson-White syndrome and likely explains the patient’s palpitations. The presence of a left bundle–appearing delta wave/QRS complex (large R in the lateral precordial leads, I, and aVL) localizes this accessory tract to the right ventricle. Another clue is the exceedingly short PR interval; the ventricle is very preexcited as the sinus impulse arrives at the right ventricle very early because the accessory tract is so close to the SA node. Left-sided bypass tracts tend to appear slightly less preexcited (somewhat longer PR interval and slightly less wide QRS). Also, more tellingly, left-sided accessory tracts have a right bundle–like appearance with negative QRS complexes in the lateral precordial leads and lead I. Septal bypass tracts can be difficult to localize and are hallmarked by a rapid transition in QRS vector between the anterior precordial leads (V1 and V2 most often).
++
++
++
A 68-year-old man with a history of myocardial infarction and congestive heart failure is comfortable at rest. However, when walking to his car, he develops dyspnea, fatigue, and sometimes palpitations. He must rest for several minutes before these symptoms resolve. Which of the following is his New York Heart Association classification?
++
++
++
++
++
++
++
++
++
+
++
The answer is C. (Chap. 279) The New York Heart Association (NYHA) classification is a tool to define criteria that describe the functional ability and clinical manifestations of patients in heart failure. It is also used in patients with pulmonary hypertension. These criteria have been shown to have prognostic value with worsening survival as class increases. They are also useful to clinicians when reading studies to understand the entry and exclusion criteria of large clinical trials. Class I is used for patients with no limiting symptoms; class II for patients with slight or mild limitation; class III implies no symptoms at rest but dyspnea, angina, or palpitations with little exertion, and patients are moderately limited; class IV is severely limited, so that even minimal activity causes symptoms. Treatment guidelines also frequently base recommendations on these clinical stages. This patient has symptoms with mild exertion but is comfortable at rest; therefore, he is NYHA class III.
++
++
++
A 47-year-old postmenopausal woman is seen for onset of severe dyspnea over the last few weeks. She reports no preceding chest pain, cough, sputum, or fever, although she does report leg swelling. Physical examination is notable for a blood pressure of 145/78 mmHg and heart rate of 123 bpm. Exophthalmos is present, as well as bilateral inspiratory crackles occupying approximately one-third of the lower chest with neck vein distention. She has a third heart sound with no murmur. Bilateral lower extremity edema is also present, with warm extremities and a fine hand tremor. Which of the following is the most likely pathophysiologic explanation for her heart failure?
++
++
++
A. Anemia with high-output state
++
++
B. Chronic systemic hypertension with resultant left ventricular hypertrophy and nonsystolic heart failure
++
++
C. Hemochromatosis with subsequent restrictive cardiomyopathy
++
++
D. Myocardial infarction with depressed left ventricular systolic function
++
++
E. Thyrotoxicosis with high-output state
+
++
The answer is E. (Chap. 279) The patient presents with evidence of heart failure by history, and physical examination confirms this diagnosis. The warm extremities make a high-output state more likely than low cardiac output. Physical examination also shows exophthalmos and a fine tremor suggestive of hyperthyroidism. Thyrotoxicosis, along with anemia, nutritional disorders, and systemic arteriovenous shunting, can all cause high-output heart failure. The eye examination and tremor make hyperthyroidism more likely than anemia. Although systolic and diastolic dysfunction are more common causes of heart failure, disorders associated with a high-output state are often reversible, and therefore, a diagnosis should be pursued when clinical clues suggest this may be present.
++
++
++
Regarding the epidemiology and prognosis of heart failure, which of the following statements is true?
++
++
++
A. Among patients with heart failure with reduced ejection fraction, coronary artery disease is the most common cause.
++
++
B. Anemia is a frequent cause of heart failure among patients with a previously structurally normal heart.
++
++
C. Due to advances in medical and device therapy, the prognosis of all patients with heart failure is now excellent, with >90% surviving more than 1 year after diagnosis.
++
++
D. Familial, or genetic, cardiomyopathy is quite rare, although it should be screened for aggressively to aid diagnosis and early treatment among presymptomatic affected family members.
++
++
E. Heart failure with preserved ejection fraction makes up a minority of the total population of patients with heart failure.
+
++
The answer is A. (Chap. 279) Coronary artery disease remains the leading cause of heart failure with reduced ejection fraction, accounting for >60% of cases. Among all patients with heart failure, approximately 50% will have a preserved ejection fraction (LV ejection fraction [LVEF] >50%; option E). Although medical and device therapy has greatly advanced in the preceding decades, prognosis for heart failure patients remains grim. Community studies have shown that 30%–40% of patients die within 1 year of diagnosis and 60%–70% die within 5 years. This prognosis becomes even more dire in patients with extreme exertional limitation. NYHA class IV patients have a 50%–70% 1-year mortality (option C). Increasingly, epidemiologic data are showing that genetic cardiomyopathy (or familial cardiomyopathy) is more common than previously believed. It is now thought that >20% of all nonischemic cardiomyopathies are genetic or familial in etiology (option D). Although anemia is often an exacerbating factor in patients who already have heart failure, it is quite rare for it to cause de novo heart failure in a patient with a structurally normal heart (option B).
++
++
++
From a pathophysiologic perspective, all of the following are upregulated in heart failure with reduced ejection fraction EXCEPT:
++
++
++
++
++
B. B-type natriuretic peptide (BNP)
++
++
C. Calcium uptake into the sarcoplasmic reticulum
++
++
++
++
+
++
The answer is C. (Chap. 279) In the failing heart, multiple systems are upregulated to attempt to maintain cardiac output through augmenting sodium and water retention (angiotensin II, aldosterone, arginine vasopressin) and myocardial contractility (norepinephrine). Although initially helpful in maintaining cardiac output, these adaptations become deleterious in contributing to the development of adverse myocardial remodeling and the congestive state. Also deleterious is the upregulation of inflammatory systems (including tumor necrosis factor). Counterregulatory natriuretic peptides (such as B-type natriuretic peptide [BNP]) are also upregulated and aid in reducing systemic vascular resistance and natriuresis. In the state of heart failure, transcriptional and posttranscriptional changes in the myocyte lead to calcium leakage through the sarcoplasmic reticular membrane and, therefore, decreased calcium uptake in the sarcoplasmic reticulum.
++
++
++
All of the following clinical conditions are associated with orthopnea EXCEPT:
++
++
++
++
++
++
++
C. Diaphragmatic weakness
++
++
++
++
E. Hepatopulmonary syndrome
+
++
The answer is E. (Chap. 279) Orthopnea, which is defined as dyspnea occurring in the recumbent position, is usually a later manifestation of heart failure than is exertional dyspnea. It results from redistribution of fluid from the splanchnic circulation and lower extremities into the central circulation during recumbency, with a resultant increase in pulmonary capillary pressure. Nocturnal cough is a common manifestation of this process and a frequently overlooked symptom of heart failure. Orthopnea generally is relieved by sitting upright or sleeping with additional pillows. Although orthopnea is a relatively specific symptom of heart failure, it may occur in patients with abdominal obesity or ascites and in patients with pulmonary disease whose lung mechanics favor an upright posture (such as with respiratory muscle compromise). In contrast, the hepatopulmonary syndrome is characterized by the development of pulmonary arteriovenous malformations and thus multiple shunts within the pulmonary circulation. These tend to develop in the lung bases, and thus, the patient’s shunt fraction increases with an upright posture. This leads to orthodeoxia (hypoxia in the upright position) and platypnea (the sensation of shortness of breath when upright) rather than orthopnea.
++
++
++
A 67-year-old man presents to the emergency department with shortness of breath. He is found to have an elevated serum BNP. In addition to heart failure, all of the following may also cause an elevation in BNP EXCEPT:
++
++
++
++
++
++
++
++
++
D. Pulmonary embolism with right heart strain
++
++
+
++
The answer is C. (Chap. 279) BNP and its NT terminal fraction are increased in heart failure states and, on the population level, contribute to prognosis in heart failure states. However, it is important to recognize that natriuretic peptide levels increase with age and renal impairment, are more elevated in women, and can be elevated in right heart failure from any cause. Levels can be falsely low in obese patients. Other biomarkers such as soluble ST-2 and galectin-3 are newer biomarkers that can be used to determine the prognosis of heart failure patients.
++
++
++
You are taking care of a patient with cor pulmonale in the medical intensive care unit. Unfortunately, he suffered a respiratory arrest at home and requires intubation and mechanical ventilation. Currently, with a tidal volume of 500 mL, FiO2 of 0.4, positive end-expiratory pressure of 20 mmHg, and respiratory rate of 20, his pH is 7.40, PCO2 is 40 mmHg, and oxygen saturation is 86%. You are concerned about the afterload experienced by his right ventricle. All of the following are likely to increase his right ventricular afterload EXCEPT:
++
++
++
A. Increasing FiO2 to increase arterial oxygen saturation to 95%
++
++
B. Increasing PEEP to 35 mmHg
++
++
C. Increasing tidal volume to 750 mL
++
++
D. Reducing respiratory rate to 10
+
++
The answer is A. (Chap. 279) The right heart is a thin-walled compliant chamber that is better suited to handle volume overload than pressure overload. In the setting of acute pressure overload (massive pulmonary embolism), the right heart quickly dilates and fails, leading to circulatory collapse. In the setting of slowly developing pulmonary hypertension (due to vascular, thrombotic, or pulmonary disease), the right heart hypertrophies in an attempt to compensate. Over time, the right ventricular (RV) contractile function becomes uncoupled from an ever-increasing afterload, and cor pulmonale or RV failure develops. In a patient with preexisting cor pulmonale, small changes in RV preload or afterload can be catastrophic. In this patient, any change leading to acidemia (reducing respiratory rate) or elevated alveolar pressures (increasing positive end-expiratory pressure, increasing tidal volume) are likely to increase RV afterload and may worsen RV stroke volume and cardiac output. Incidentally, increasing transmural pressure (plateau or end-expiratory ventilator pressures) has disparate effects on RV and LV afterload. Because the RV target vasculature is completely intrathoracic, the pressure is transmitted across the vessel walls and must be overcome by the RV. However, LV vasculature is mostly extrathoracic. Thus, the increased intrathoracic pressure is only transmitted to the LV and actually reduces afterload (transmural chamber pressure). Hypoxia causes pulmonary vasoconstriction (as opposed to the systemic circulation, where it causes vasodilation). Thus, increasing the FiO2 in an attempt to improve the hypoxia may reduce the pulmonary vasoconstriction and lower the RV afterload.
++
++
++
Mr. George is a 52-year-old man with longstanding hypertension and poorly controlled diabetes. He presents complaining of several months of breathlessness with exertion, acute episodes of shortness of breath when recumbent, and lower extremity edema. On examination, he has an elevated jugular venous pulse and an S4 on auscultation. Echocardiography shows a left ventricular ejection fraction of 55% with a large left atrium. You suspect this patient has the syndrome of heart failure with preserved ejection fraction (HFpEF). Drugs targeting which of the following have convincingly demonstrated mortality reduction for patients with HFpEF?
++
++
++
A. Angiotensin-converting enzyme
++
++
++
++
++
++
D. Sodium-potassium-ATPase
++
++
+
++
The answer is E. (Chap. 280) While therapeutic targets in heart failure with reduced ejection fraction (HFrEF) are relatively abundant and guided by the targets of disease modification, trials in heart failure with preserved ejection fraction (HFpEF) have been largely disappointing. ACE inhibitors (option A) have been studies in many mechanistic studies and have shown no convincing mortality benefit. Similarly, angiotensin receptor blockers (option B) were studied in the CHARM-Preserved and I-PRESERVE studies and were found to have no mortality benefit. Sildenafil (a phosphodiesterase-5 inhibitor) was shown to improve filling pressures and RV function in patients with HFpEF, but did not show mortality benefit. Finally, digoxin, which inhibits sodium-potassium-ATPase, was shown to have no role in treating HFpEF in the DIG study. Overall, symptom management and blood pressure control are the goals in HFpEF treatment now. One should also remain vigilant for myocardial ischemia.
++
++
++
You are evaluating a 64-year-old woman with a history of nonischemic cardiomyopathy. She presents to the emergency department for shortness of breath. You note that she has gained 11 kg since her last cardiology appointment 2 months ago. Physical examination confirms findings associated with acute decompensated heart failure, including pulmonary rales, elevated jugular venous pulse, abdominal ascites, lower extremity edema, and a square wave blood pressure Valsalva response. Her extremities are warm, and blood pressure is 110/78 mmHg with a heart rate of 75 bpm. Her laboratory studies return with a sodium of 128 mEq/L and creatinine of 2.5 mg/dL (which is increased from her prior level of 1.2 mg/dL). Chest x-ray shows a diffuse alveolar filling pattern consistent with pulmonary edema. What is the next most appropriate step?
++
++
++
A. Administer digoxin 250 μg IV
++
++
B. Administer furosemide 40 mg IV
++
++
C. Insert a large-bore central IV line and prepare for ultrafiltration
++
++
D. Insertion of a pulmonary artery catheter for hemodynamic monitoring
++
++
E. Start dobutamine at 5 μg/kg/min and titrate to a urine output of 1 mL/kg/hr
+
++
The answer is B. (Chap. 280) This patient clearly has adequate cardiac output to maintain peripheral perfusion as evidenced by her physical examination (warm extremities) and adequate blood pressure. However, her elevated creatinine is vexing and indicates that she suffers from the cardiorenal syndrome. In some cases, it truly is a depressed cardiac output causing a low glomerular filtration rate (GFR); however, these cases are typically accompanied by other signs of peripheral malperfusion. In most cases when cardiac output is not severely depressed, it is thought that a complex interplay of elevated venous pressures (reducing transglomerular perfusion pressures) and abdominal pressures leads to decreased GFR. In these cases, reducing venous pressures with diuretics is the most reasonable first option. In patients who respond poorly (rising creatinine or adverse hemodynamic effects), hemodynamic monitoring (option A) or ultrafiltration can be considered. Digoxin should be used with caution in renal insufficiency and has no real benefit acutely here. In cases where cardiac output is thought to be severely depressed and peripheral perfusion is compromised, inotropic therapy may be indicated.
++
++
++
Which of the following statements regarding nesiritide is true in patients with acute decompensated heart failure requiring invasive hemodynamic monitoring?
++
++
++
A. Nesiritide has no significant effect on blood pressure.
++
++
B. Nesiritide improves renal perfusion in states of acute decompensated heart failure.
++
++
C. Nesiritide is associated with a decreased rate of rehospitalization, but not death, from acute decompensated heart failure.
++
++
D. Nesiritide reduces pulmonary capillary wedge more rapidly than IV nitrates.
++
++
E. None of the statements are true.
+
++
The answer is D. (Chap. 280) Nesiritide is the recombinant form of human BNP. It was introduced in a fixed dose for therapy of acute decompensated heart failure after demonstration of a more rapid and greater reduction in pulmonary capillary wedge pressure than with intravenous nitrates. Enthusiasm for nesiritide waned due to concerns within the pivotal trials for development of renal insufficiency and an increase in mortality. To address these concerns, a large-scale morbidity and mortality trial, the Acute Study of Clinical Effectiveness of Nesiritide in Decompensated Heart Failure (ASCEND-HF), was completed in 2011 and randomly enrolled 7141 patients with acute decompensated heart failure to nesiritide or placebo for 24–168 hours in addition to standard care. Nesiritide was not associated with an increase or a decrease in the rates of death and rehospitalization and had a clinically insignificant benefit on dyspnea. Renal function did not worsen, but increased rates of hypotension were noted. Although this trial established the safety of nesiritide, its routine use cannot be advocated due to lack of significant efficacy.
++
++
++
Mr. Jones is a 21-year-old man who presents to the emergency department with several days of worsening shortness of breath and lethargy 1 week after a viral upper respiratory tract infection. His family brought him in after he lacked the energy to rise from the couch unassisted. His blood pressure is 88/72 mmHg, heart rate is 115 bpm, and room air oxygen saturation is 90%. Physical examination reveals pulmonary crackles, elevated jugular venous pressure, an audible S3 gallop, and cool extremities. He is lethargic and slow to respond to questions. Laboratory analysis reveals a creatinine of 2.3 mg/dL, elevated BNP level, and mildly elevated lactate. Bedside echocardiography reveals a left ventricular ejection fraction of 15% with global hypokinesis. You start dobutamine at 5 μg/kg/min and prepare to insert a pulmonary artery catheter for hemodynamic monitoring. Which of the following hemodynamic parameters is most likely increased?
++
++
++
++
++
B. Left ventricular stroke work index
++
++
C. Mixed venous oxygen saturation
++
++
++
++
E. Systemic vascular resistance
+
++
The answer is E. (Chaps. 279 and 280) The utility of invasive hemodynamic monitoring during acute decompensated heart failure has been highly scrutinized recently. Based on several observational and randomized trials, the routine use of a pulmonary artery catheter is not recommended and should be restricted to patients who respond poorly to diuresis or experience hypotension or signs and symptoms suggestive of a low cardiac output where therapeutic targets are unclear. In this patient with hypotension and signs of low cardiac output, invasive monitoring will allow the clinician to rapidly, objectively assess any changes in hemodynamic status and respond appropriately. In states of “cold” (poor perfusion) and “wet” (elevated filling pressures, or hypervolemia) heart failure, the stroke volume and cardiac output are decreased. Left ventricular stroke work index (a calculated value that normalizes left ventricular work for the patient’s body surface area and afterload) is also diminished. Mixed venous oxygen saturation is greatly diminished as well, as the Fick equation dictates that cardiac output is proportional to the venous oxygen saturation if oxygen consumption and arterial oxygen saturation are normal. Systemic vascular resistance equals the mean systemic arterial pressure minus the mean right atrial pressure divided by cardiac output. As cardiac output drops, the systemic vasculature will increase its resistance to attempt to maintain blood pressure and end-organ perfusion pressure. However, this initially compensatory action becomes deleterious because an inefficient left ventricle must work against an ever-increasing afterload. The most effective inotropic agents in acute decompensated heart failure (milrinone, and dobutamine) also have vasodilatory properties to combat this harmful systemic vascular resistance (SVR) increase.
++
++
++
Which of the following medications has been shown to reduce mortality in heart failure with reduced ejection fraction?
++
++
++
++
++
++
++
++
++
++
++
+
++
The answer is C. (Chap. 280) β-Blockers represent the most-studied drug in the history of HFrEF. Initially avoided for their negative inotropic and chronotropic effects, they now form the foundation of neurohormonal-targeted therapy in the treatment of chronic HFrEF. However, the benefit of β-blockers is not a blanket class effect and indeed has only been demonstrated for three drugs in the numerous clinical trials: metoprolol succinate, bisoprolol, and carvedilol. Importantly, metoprolol tartrate has never been found to have mortality benefit in HFrEF, nor have β-blockers with intrinsic sympathomimetic activity (nebivolol, xamoterol).
++
++
++
You are seeing a patient with newly diagnosed nonischemic cardiomyopathy with a left ventricular ejection fraction of 35%. He is euvolemic and New York Heart Association (NYHA) class I in terms of symptoms severity. His blood pressure is 120/78 mmHg, and heart rate is 80 bpm. You know that, eventually, you would like him on optimal doses of a β-blocker and an ACE inhibitor. Which of the following represents the appropriate order of initiation and dose titration of these medications?
++
++
++
A. Co-initiate ACE inhibitor and β-blocker at low dose. Titrate each up every 2–4 weeks to highest tolerated dose.
++
++
B. Initiate ACE inhibitor and titrate to highest tolerated dose. Then initiate β-blocker and titrate to highest tolerated dose
++
++
C. Initiate ACE inhibitor at low dose. Then initiate β-blocker at low dose. Alternate titrating the dose up for each drug to the highest tolerated dose for both.
++
++
D. Initiate β-blocker and titrate to highest tolerated dose. Then initiate ACE inhibitor and titrate to highest tolerated dose
++
++
E. The specific strategy does not matter so much as the goal of initiating both ACE inhibitor and β-blocker in a timely manner and titrating both to their optimal, highest tolerated doses.
+
++
The answer is E. (Chap. 280) Whether β-blockers or ACE inhibitors should be started first was answered by the Cardiac Insufficiency Bisoprolol Study (CIBIS) III, in which outcomes did not vary when either agent was initiated first. Thus, it matters little which agent is initiated first; what does matter is that optimally titrated doses of both ACE inhibitors and β-blockers are established in a timely manner. It should be noted that although ACE inhibitors have been shown to have a dose-dependent reduction in hospitalizations, higher tolerated doses do not materially improve survival. In contrast, β-blockers have a strong dose effect on survival. Thus, if forced to choose between a higher dose of β-blockers or ACE inhibitor due to low blood pressure, the higher dose of β-blocker may provide more survival benefit.
++
++
++
You are seeing a 72-year-old woman with a history of ischemic cardiomyopathy and left ventricular ejection fraction of 35%. She is NYHA class II in terms of symptoms. She is on lisinopril 40 mg daily, carvedilol 25 mg twice daily, and spironolactone 25 mg daily. Her blood pressure is 102/82 mmHg, and heart rate is 60 bpm. Which of the following additional medications will further reduce her mortality?
++
++
++
++
++
++
++
++
++
++
++
+
++
The answer is E. (Chap. 280) This patient is on optimal doses of the three medications forming the foundation of neurohormonal therapy for HFrEF. Digoxin was shown to reduce hospitalizations but not mortality in the DIG trial. The addition of valsartan to ACE inhibitor therapy was studied in the Val-HEFT trial and showed a trend toward worse outcomes. Similarly, aliskiren (a direct renin inhibitor) was studied in addition to ACE inhibitors in the ASTRONAUT trial and showed no mortality benefit and an abundance of side effects such as hyperkalemia and hypotension. Ivabradine, a novel heart rate–reducing agent, was studied in the SHIFT trial and showed mortality benefit, but only in patients with a heart rate >70 bpm already on βblockers. This patient’s heart rate is controlled.
++
++
++
You are following a 57-year-old man who suffered a large anterior myocardial infarction 8 months ago. Despite a late presentation to medical care (>5 hours after chest pain onset), he underwent primary coronary intervention with excellent angiographic result. After that stay in the hospital, he has been followed by you regularly and maintains excellent medication compliance. Today, his heart rate is 65 bpm, and blood pressure is 104/82 mmHg. He looks quite well and reports absolutely no symptoms, even during a recent elk hunting trip requiring long days of hiking up and down hills. His medications are aspirin, clopidogrel, atorvastatin, carvedilol, and valsartan. His ECG shows sinus rhythm, with a right bundle branch block and QRS duration of 135 msec. Echocardiography today shows an ejection fraction of 25% with a large area of anterior dyskinesis consistent with aneurysm. Which of the following therapies is indicated to reduce this patient’s mortality?
++
++
++
A. Implantable cardioverter defibrillator
++
++
B. Implantable cardioverter defibrillator with cardiac resynchronization capability (coronary sinus lead)
++
++
++
++
D. Surgical ventricular remodeling
++
++
+
++
The answer is A. (Chap. 280) Sudden cardiac death due to ventricular arrhythmias is the mode of death in approximately half of patients with heart failure and is particularly proportionally prevalent in patients diagnosed with HFrEF in early stages of the disease. Although primary prevention of sudden cardiac death is challenging, the two most important risk markers for increased risk of death are the degree of residual left ventricular dysfunction despite optimal medical therapy (≤35%) and the underlying etiology of heart failure (post–myocardial infarction or ischemic cardiomyopathy). Currently, patients with NYHA class II to III symptoms of heart failure and an LVEF <35%, irrespective of etiology of heart failure, are appropriate candidates for implantable cardioverter defibrillator (ICD) prophylactic therapy. In patients with a history of myocardial infarction and optimal medical therapy with residual LVEF ≤30% (even when asymptomatic), placement of an ICD is appropriate. The Resynchronization–Defibrillation for Ambulatory Heart Failure Trial (RAFT) and Multicenter Automatic Defibrillator Implantation Trial with Cardiac Resynchronization Therapy (MADIT-CRT) both sought to use cardiac resynchronization therapy (CRT) in combination with an ICD. Most benefit in mildly symptomatic HFrEF patients accrues from applying this therapy in those with a QRS width of >149 msec and a left bundle branch block pattern. Attempts to further optimize risk stratification and expand indications for CRT using modalities other than electrocardiography have proven disappointing. Likewise, surgical ventricular remodeling was studied in a 1000-patient trial and found to have no disease-modifying effect. It is still used, but relegated to patients with cardioembolic strokes on anticoagulation or patients with refractory ventricular arrhythmias. Ivabradine, a novel heart rate–reducing agent, may reduce mortality in patients with HFrEF who have a resting heart rate >70 bpm while on an adequate dose of a β-blocker.
++
++
++
You are evaluating a 25-year-old man with a history of nonischemic cardiomyopathy who underwent an orthotopic heart transplant 18 months ago. He has done very well with his course and comes today for a routine visit. You note that his blood pressure is 128/82 mmHg and heart rate is 90 bpm. His ECG appears typical for sinus rhythm with a normal PR interval and P-wave axis. On review of his chart, you note that his resting heart rate has been almost exactly 90 bpm on every visit. He feels fine. Which of the following is the most appropriate next step for his elevated resting heart rate?
++
++
++
A. Exercise treadmill ECG testing to evaluate for chronotropic competence
++
++
B. Initiate diltiazem 120 mg twice daily
++
++
C. Initiate metoprolol tartrate 25 mg twice daily
++
++
++
++
E. Refer for electrophysiology study to evaluate for ectopic atrial tachycardia
+
++
The answer is D. (Chap. 281) In patients with an orthotopic heart transplant, the donor heart is denervated during the harvest. In the absence of sympathetic or parasympathetic innervation, many patients exhibit a mild resting tachycardia or slightly elevated heart rate. Heart rate response during exercise or physiologic stress may be delayed, but is enacted by cardiac response to circulating catecholamines, primarily adrenal in origin. It is important to realize this to avoid unnecessary testing and to avoid adverse events with drug administration. In particular, atropine will have almost no effect on a patient after orthotopic heart transplant, and extreme caution should be exercised in administering adenosine because the denervated heart may have a very delayed recovery from adenosine-induced heart block.
++
++
++
You are appointed to an advisory position with the government’s committee on organ transplantation. The committee chairman is curious about enacting a nationwide organ-sharing strategy for heart transplant, wherein only position on the transplant list and wait time would determine organ allocation order (instead of any geographic concerns). You advise the committee chairman that this strategy is not beneficial because:
++
++
++
A. Blood types are grouped nonrandomly nationally, precluding true nationwide sharing
++
++
B. The different regions have highly variable criteria for severity of illness determination
++
++
C. The financial structure of reimbursement/payment nationwide would not allow it
++
++
D. There is a physiologic limit on “cold ischemic” (time out of body) time for harvested hearts that would preclude nationwide sharing
++
++
E. None of the above are true
+
++
The answer is D. (Chap. 281) In the United States, the allocation of donor organs is accomplished under the supervision of the United Network for Organ Sharing, a private organization under contract to the federal government. The United States is divided geographically into 11 regions for donor heart allocation. Allocation of donor hearts within a region is decided according to a system of priority that takes into account (1) the severity of illness, (2) the geographic distance from the donor, and (3) the patient’s time on the waiting list. A physiologic limit of ~3 hours of “ischemic” (out-of-body) time for hearts precludes a national sharing of hearts. This allocation system design is reissued annually and is responsive to input from a variety of constituencies, including both donor families and transplantation professionals.
++
++
++
A 28-year-old man presents to the emergency department for dyspnea on exertion. He had an orthotopic heart transplant for non-ischemic cardiomyopathy 5 years ago and, in general, has done quite well except for one cytomegalovirus reactivation within the first year. He reports that for the past 3 months, he has noticed that with decreasing amounts of exertion he has been having limiting dyspnea. He is adamant that he is experiencing no chest pain or pressure during these episodes. He has been perfectly compliant with his regimen of tacrolimus, mycophenolate mofetil, and low-dose prednisone. Echocardiography reveals a normal LV function with normal LV wall thickness. His resting ECG shows normal sinus rhythm at a rate of 80 bpm. Which of the following is the most likely cause of his symptoms?
++
++
++
A. Antibody-mediated rejection
++
++
++
++
C. Coronary artery disease
++
++
++
++
E. Medication side effect
+
++
The answer is C. (Chap. 281) Despite usually having young donor hearts, cardiac allograft recipients are prone to develop coronary artery disease (CAD). This CAD is generally a diffuse, concentric, and longitudinal process that is quite different from “ordinary” atherosclerotic CAD, which is more focal and often eccentric. The underlying etiology most likely is primarily immunologic injury of the vascular endothelium, but a variety of risk factors influence the existence and progression of CAD, including nonimmunologic factors such as dyslipidemia, diabetes mellitus, and cytomegalovirus (CMV) infection (as in this patient). It is hoped that newer and improved immunosuppressive modalities will reduce the incidence and impact of these devastating complications, which currently account for the majority of late posttransplantation deaths. Thus far, the immunosuppressive agents mycophenolate mofetil and the mammalian target of rapamycin (mTOR) inhibitors sirolimus and everolimus have been shown to be associated with short-term lower incidence and extent of coronary intimal thickening; in anecdotal reports, institution of sirolimus was associated with some reversal of CAD. The use of statins also is associated with a reduced incidence of this vasculopathy, and these drugs are now used almost universally in transplant recipients unless contraindicated. Palliation of CAD with percutaneous interventions is probably safe and effective in the short term, although the disease often advances relentlessly. Because of the denervated status of the organ, patients rarely experience angina pectoris, even in advanced stages of disease. Antibody (humoral)–mediated rejection is exceedingly rare in a patient this far out from transplant, particularly one who fortunately has had no prior rejection episodes. While cellular rejection is possible, the presence of a normal left ventricle (and no arrhythmias) makes it less likely. Tacrolimus most commonly causes hypertension, neurologic complications, and renal insufficiency. Mycophenolate mofetil most commonly causes bone marrow suppression and diarrhea. Low-dose prednisone is generally well tolerated, although chronic steroids carry incipient risk of diabetes, obesity, skin changes, iatrogenic adrenal insufficiency, osteoporosis, and cataracts. None of these drugs classically causes dyspnea.
++
++
++
You are seeing Mrs. Block in the heart failure clinic. She is a 68-year-old woman with a past history of diffuse large B-cell lymphoma treated successfully with chemotherapy and eventual bone marrow transplantation 15 years previously. Unfortunately, she suffered chemotherapy-related cardiomyopathy and now has an ejection fraction of 15%. She has been hospitalized four times in the last 6 months with acute decompensated heart failure. She is currently NYHA class III to IV regarding symptoms. Recently, she underwent a cardiopulmonary exercise stress test documenting a peak oxygen consumption of 9 mL/kg/min. She is on optimal medical therapy. At a multidisciplinary transplantation meeting, she was deemed to not be a candidate for orthotopic heart transplantation due to her age and prior malignancy. She asks what other options are available for her, as her number one priority is to live 3 more years to see her grandson graduate high school. Which of the following is an appropriate response?
++
++
++
A. “Continuous infusion of milrinone will improve cardiac function and survival.”
++
++
B. “A left ventricular assist device can improve quality of life but provides no survival benefit over optimal medical care.”
++
++
C. “A left ventricular assist device can improve survival over the initial 2–5 years more than optimal medical therapy.”
++
++
D. “Stem cell implantation has been shown to prolong life in patients with end-stage cardiomyopathy.”
++
++
E. “There are no other options for prolonging life, and we should consider palliative measures in the future.”
+
++
The answer is C. (Chap. 281) For patients with end-stage cardiomyopathy (NYHA class IV or peak oxygen consumption <14 mL/kg/min), the prognosis is abysmal. In the landmark REMATCH trial, the 2-year survival of the medically treated arm was only 8%. Orthotopic heart transplantation is the gold standard therapy for these patients. However, some patients are not candidates due to underlying comorbidities that would render a transplantation too dangerous. For these patients, the continuous flow left ventricular assist devices have been shown convincingly to improve mortality (2-year survival in REMATCH was approximately 60% and has improved since that study). Although it is unreasonable to predict a complication-free course (most patients have thrombotic, infectious, or neurologic complications during their course of support), the median survival for patients with left ventricular assist device support is now approaching 5 years. Stem cell therapy is currently investigational only and has never been shown to meaningfully impact survival. Similarly, milrinone or dobutamine therapy (inotropic support) can be used continuously to support quality of life as a palliative goal or as a bridge to initiation of mechanical support or transplantation. However, all studies show that the use of inotropes negatively impacts survival.
++
++
++
Which of the following statements regarding the epidemiology of congenital heart disease (CHD) in the United States is true?
++
++
++
A. CHD remains extremely rare, complicating 0.1% of all live births.
++
++
B. Given advances in surgical techniques and pre- and postnatal care, survival for a neonate with CHD now approaches 90%.
++
++
C. Given the declining rates of pregnancy in women with CHD, the incidence of CHD in neonates is declining.
++
++
D. The population of adults with CHD is declining given improved prenatal screening efforts.
++
++
E. Women with CHD are at no increased risk for complications during pregnancy compared with the normal population.
+
++
The answer is B. (Chap 282) The most common birth defects are cardiovascular in origin. These malformations are due to complex multifactorial genetic and environmental causes. Recognized chromosomal aberrations and mutations of single genes account for <10% of all cardiac malformations. Congenital heart disease (CHD) complicates ~1% of all live births in the general population—about 40,000 births per year—but occurs more frequently in the offspring (about 4%–10%, depending on maternal CHD type) of women with CHD. Due to the remarkable surgical advances over the last 60 years, >90% of afflicted neonates and children now reach adulthood; women with CHD may now frequently successfully bear children after competent repairs. As such, the population with CHD is steadily increasing. Women with CHD are at increased risk for peri- and postpartum complications, but maternal CHD is generally not considered an absolute contraindication to pregnancy unless the mother has certain high-risk features (e.g., cyanosis, pulmonary hypertension, decompensated heart failure, arrhythmias, aortic aneurysm).
++
++
++
A 62-year-old man is being evaluated for mitral valve replacement surgery for severe mitral regurgitation. As part of his evaluation, he undergoes a transesophageal echocardiogram that demonstrates a small jet of right-to-left Doppler flow during systole across the atrial septum. The jet is located roughly in the middle of the septum and occurs when a small flap of tissue swings open <1 mm. There is no diastolic flow, nor is there a visible opening in any part of the septum during diastole. Which of the following explains the finding on echocardiography?
++
++
++
A. Ostium primum atrial septal defect
++
++
B. Ostium secundum atrial septal defect
++
++
C. Partial anomalous pulmonary venous return
++
++
++
++
E. Sinus venosus atrial septal defect
+
++
The answer is D. (Chap. 281) Atrial septal defect (ASD) is a common cardiac anomaly that may be first encountered in the adult and occurs more frequently in females. Sinus venosus ASD occurs high in the atrial septum near the entry of the superior vena cava into the right atrium and is associated frequently with anomalous pulmonary venous connection from the right lung to the superior vena cava or right atrium. Ostium primum ASDs lie adjacent to the atrioventricular valves, either of which may be deformed and regurgitant. Ostium primum ASDs are common in Down syndrome, often as part of complex atrioventricular septal defects with a common atrioventricular valve and a posterior defect of the basal portion of the interventricular septum. The most common ostium secundum ASD involves the fossa ovalis and is mid-septal in location; this should not be confused with a patent foramen ovale (which is present in ~25% of healthy adults). Anatomic obliteration of the foramen ovale ordinarily follows its functional closure soon after birth, but residual “probe patency” is a common normal variant; ASD denotes a true deficiency of the atrial septum and implies functional and anatomic patency.
++
++
++
You are evaluating a 21-year-old man in clinic. He occasionally feels tired during his senior year of college but is otherwise asymptomatic. You first saw him 6 weeks ago and noted a harsh, holosystolic murmur at the left lower sternal border that augmented with hand grip. Suspecting a ventricular septal defect (VSD), you referred him for echocardiography, which confirmed your suspicion, visualizing a 5-mm VSD in the muscular interventricular septum. Subsequent right heart catheterization for hemodynamic assessment revealed a mean pulmonary artery pressure of 20 mmHg, pulmonary venous resistance of 2 Wood units, and systemic vascular resistance of 6 Wood units. Through meticulous serial measurements of venous oxygen saturations in the central veins and right heart, you calculate a right heart cardiac output of 7.5 L/min and systemic cardiac output of 6 L/min. Given these findings, you recommend what therapeutic course?
++
++
++
A. Cardiopulmonary exercise testing to define peak oxygen consumption
++
++
B. Closure with a percutaneously deployed septal occluder device
++
++
C. Consideration of heart and lung transplant
++
++
D. No intervention now; annual cardiology assessments and reimaging if any new symptoms emerge
++
++
E. Surgical correction via open heart surgery and patch repair
+
++
The answer is D. (Chap. 281) This patient has a small muscular VSD. The decision on whether to treat a VSD is complex but is based on the principle of avoiding any right ventricular and pulmonary vascular compromise. Similarly, one must avoid repairing a VSD in the setting of overt pulmonary hypertension, as this is clearly associated with worse outcomes. Closure is not recommended for patients with normal pulmonary arterial pressures with small shunts (pulmonary-to-systemic flow ratios of <1.5:1). Operative correction or transcatheter closure is indicated when there is a moderate to large left-to-right shunt with a pulmonary-to-systemic flow ratio >1.5:1, in the absence of prohibitively high levels of pulmonary vascular resistance (pulmonary arterial resistance is less than two-thirds of systemic arterial resistance). This patient has a normal pulmonary artery pressure, a pulmonary vascular resistance to SVR ratio of 0.33, and Qp:Qs of 1.25; therefore, closure is not warranted at this time, but follow-up is indicated.
++
++
++
You are evaluating a 52-year-old patient who presented to the emergency department for chest pain and elevated cardiac enzymes. On auscultation, you can easily hear a superficial, continuous murmur at the mid-sternal level that was never documented before. Echocardiography is limited but is able to visualize all the cardiac valves, which appear completely normal. The patient undergoes a right heart catheterization. The oxygen saturation values are shown below. What is the abnormality accounting for this patient’s symptoms?
++
++
++
++
A. Anomalous left coronary artery off the main pulmonary artery
++
++
++
++
C. Coronary arteriovenous fistula to the coronary sinus
++
++
D. Patent ductus arteriosus
++
++
E. Ventricular septal defect
+
++
The answer is C. (Chap. 281) This patient has the classic murmur and findings of myocardial ischemia associated with a spontaneous rupture of a coronary artery aneurysm to form a coronary arteriovenous fistula. In this case, the presence of an oxygenation step-up in the right heart, with near-systemic oxygenation levels in the coronary sinus (which usually has very low oxygen saturation levels), makes this diagnosis certain. An ASD would not present with such a murmur or clinical presentation. A VSD would not show such high oxygen levels in the coronary sinus. A patent ductus arteriosus can present with a similar murmur but usually causes left-to-right shunting at the level of the pulmonary artery; thus, the oxygen step-up would be there. Anomalous left coronary artery off the pulmonary artery causes myocardial ischemia, although not so precipitously and usually early in life, and is not associated with a murmur. In this condition, oxygenated blood from the aortic root flows via a dilated right coronary artery and collaterals to the left coronary artery and retrograde to the lower pressure pulmonary artery circulation via the anomalous left main coronary artery (which emerges from the pulmonary artery). Most patients die in the first year of life from myocardial ischemia and fibrosis, although a minority can live into adulthood.
++
++
++
Mr. Jenson is a 40-year-old man with a congenital bicuspid aortic valve who you have been seeing for more than a decade. You obtain an echocardiogram every other year to follow the progression of his disease knowing that bicuspid valves often develop stenosis or regurgitation requiring replacement in middle age. Given his specific congenital abnormality, what other anatomic structure is important to follow on his biannual echocardiograms?
++
++
++
++
++
++
++
C. Pulmonary artery pressures
++
++
D. Pulmonic valve function
++
++
E. Tricuspid valve regurgitation
+
++
The answer is A. (Chap. 282) Bicuspid aortic valve is among the most common of congenital heart cardiac abnormalities. Valvular function is often normal in early life and thus may escape detection. Due to abnormal flow dynamics through the bicuspid aortic valve, the valve leaflets can become rigid and fibrosed, leading to either stenosis or regurgitation. However, pathology in patients with bicuspid aortic valve is not limited to the valve alone. The ascending aorta is often dilated, misnamed “poststenotic” dilatation; this is due to histologic abnormalities of the aortic media and may result in aortic dissection. It is important to screen specifically for aortopathy because dissection is a common cause of sudden death in these patients.
++
++
++
All of the following are classic definitional features of the tetralogy of Fallot EXCEPT:
++
++
++
A. Obstruction to RV outflow
++
++
++
++
++
++
++
++
E. Ventricular septal defect
+
++
The answer is D. (Chap. 282) The four classic components of the tetralogy of Fallot are malaligned VSD, obstruction to RV outflow, aortic override of the VSD, and RV hypertrophy (due to the RV’s response to aortic pressure via the large VSD). Tricuspid atresia is associated with Ebstein anomaly and hypoplastic right heart syndrome. In these cases, a concurrent systemic-to-pulmonary connection is required to maintain early life (such as a patent ductus arteriosis). Right heart bypass operations such as the Glenn shunt or Fontan palliation can provide adequate pulmonary flow to support patients into and through early adulthood.
++
++
++
A 78-year-old man is evaluated for the onset of dyspnea on exertion. He has a long history of tobacco abuse, obesity, and diabetes mellitus. His current medications include metformin, aspirin, and occasional ibuprofen. On physical examination, his peripheral pulses show a delayed peak, and he has a prominent left ventricular heave. He is in a regular rhythm with a IV/VI mid-systolic murmur, loudest at the base of the heart and radiating to the carotid arteries. A fourth heart sound is present. Echocardiography confirms severe aortic stenosis without other valvular lesions. Which of the following most likely contributed to the development of his cardiac lesion?
++
++
++
A. Congenital bicuspid aortic valve
++
++
++
++
C. Occult rheumatic heart disease
++
++
D. Underlying connective tissue disease
++
++
+
++
The answer is B. (Chap. 283) The patient has aortic stenosis that presented late in life. Although bicuspid aortic valve underlies nearly half of all aortic stenosis cases, this lesion typically presents earlier in life, and only 40% of patients >70 years old with aortic stenosis who undergo surgery have a bicuspid valve. Rheumatic heart disease may cause aortic stenosis, but nearly invariably, mitral stenosis is also present. Underlying connective tissue disease is not known to be associated with aortic stenosis. Modern research on development of aortic stenosis has shown that several traditional atherosclerotic risk factors are present such as diabetes mellitus, smoking, chronic kidney disease, and the metabolic syndrome. Polymorphisms of the vitamin D receptor have also been demonstrated in patients with symptomatic aortic stenosis.
++
++
++
A 63-year-old man presents with new-onset exertional syncope and is found to have aortic stenosis. In counseling the patient, you tell him that your therapeutic recommendation is based on the observation that untreated patients with his presentation have a predicted average life span of:
++
++
++
++
++
++
++
++
++
++
++
+
++
The answer is C. (Chap. 283) Exertional syncope is a late finding in aortic stenosis and portends a poor prognosis. Patients with this symptom or with angina pectoris have an average time to death of 3 years. Patients with dyspnea have an average time to death of 2 years, and patients with heart failure have an average time to death of 1.5–2 years. Given these data, patients with severe aortic stenosis and symptoms should strongly be considered for surgical therapy.
++
++
++
Mr. Belliard is an 82-year-old man who you previously followed in clinic. He was last seen 3 years ago, when he noted occasional syncope. Subsequent workup revealed severe calcific aortic stenosis, with an aortic valve area of 0.7 cm2 and mean gradient of 45 mmHg. At that time, you recommended surgical aortic valve replacement. On the day of his operation, however, Mr. Belliard called to say he really didn’t feel quite sick enough to undergo open heart surgery, and he’d be back in touch when he was ready. He missed all subsequent follow-up appointments. Today, he presents to the emergency department after several weeks of lethargy and dyspnea. His blood pressure is 82/68 mmHg and heart rate is 110 bpm, and he has an ECG showing sinus rhythm. His carotid impulse is weak and delayed, and extremities are cool to the touch. You again appreciate an S4 on auscultation and a III/VI late-peaking systolic murmur. His renal and liver function are now impaired, and urine output is poor. The cardiothoracic surgeon on call defers emergent surgical valve replacement due to his acute renal and hepatic injury. Which of the following would be a reasonable therapeutic option to improve Mr. Belliard’s short-term perfusion to permit aortic valve replacement?
++
++
++
++
++
B. Digoxin 250 μg IV once and 125 μg orally daily thereafter
++
++
C. Metoprolol 5 mg IV serial dosing targeting a resting heart rate of 60–70 bpm
++
++
D. Percutaneous aortic balloon valvuloplasty
++
++
E. Phenylephrine continuous IV infusion titrated to a mean arterial pressure of >65
+
++
The answer is D. (Chap 283) This patient is in cardiogenic shock due to his progressive aortic stenosis (AS). The natural history of symptomatic severe AS is not encouraging. Aortic valve area, on average, declines by 0.1 cm2 per year, and gradient increases by 7 mmHg annually. The average time to death after the onset of various symptoms is as follows: angina pectoris, 3 years; syncope, 3 years; dyspnea, 2 years; and congestive heart failure, 1.5–2 years. Moreover, in >80% of patients who die with AS, symptoms had existed for <4 years. Currently, this patient has compromised organ function due to the severe LV obstruction of AS and resultant low stroke volume. Any negative inotropic or chronotropic agents may prove fatal, as reducing heart rate or stroke volume will further compromise cardiac output (option C). Digoxin is unlikely to augment stroke volume to any significant degree in the setting of severe AS and is risky in the setting of compromised renal function (option B). Although statins have been shown to slightly slow the progression of AS, they serve no role in the acute setting (option A). Phenylephrine is an α-agonist. Although it may serve to increase blood pressure, it acts via peripheral vasoconstriction, which further increases the resistance the left ventricle is working against. Percutaneous aortic balloon valvuloplasty (PABV) is a poor long-term therapeutic option because almost all patients have the recrudescence of severe AS within 6 months to a year. However, PABV can serve as bridge to definite therapy (such as surgical aortic valve replacement), temporarily improving cardiac output and end-organ perfusion to render surgical risk acceptable. Other options would be an intra-aortic balloon pump or, in rare cases, mechanical circulatory support.
++
++
++
An 85-year-old former lawyer presents with several months of accelerating dyspnea on exertion and lower extremity edema. On examination, you note a laterally displaced point of maximal impulse (PMI) and an S4 gallop. She has a III/VI systolic murmur at the base radiating to the carotid arteries. Transthoracic echocardiogram reveals a left ventricular ejection fraction of 25% with global hypokinesis. Calculated aortic valve area is 0. 8cm2 and mean gradient is 25 mmHg. What is the next most reasonable step to determine whether this patient would benefit from aortic valve replacement?
++
++
++
A. Cardiac magnetic resonance imaging (MRI) to evaluate ventricular scar and aortic valve morphology
++
++
B. Cardiac positron emission tomography (PET) to determine ventricular viability
++
++
C. Coronary angiography to evaluate for the presence of obstructive coronary disease
++
++
D. Dobutamine stress echocardiography
++
++
E. Right heart catheterization to document cardiac output and filling pressures
+
++
The answer is D. (Chap. 283) This patient has evidence of the clinical entity referred to as low-gradient, low-flow AS. Conceptually, the aortic valve area during systole is dependent on two factors: (1) aortic valve morphology (e.g., calcific AS with restricted leaflet motion), and (2) ventricular contractile force. Even a normal aortic valve will open very little if the ventricle contracts very weakly. In this patient’s case, the finding of a low calculated aortic valve area without a severely high gradient (severe is >40 mmHg) in the setting of reduced LV function defines the entity of low-gradient, low-flow AS. It is difficult to determine whether the valve area is low due to reason 1 or 2 during a resting echocardiography. However, dobutamine stress will accomplish two goals. First, it will assess for ventricular viability (the ability to increase stroke volume by 20%), which is shown to predict outcomes after aortic valve replacement. Second, and more importantly, as the ventricular contractility increases, it allows the clinician to differentiate between true, morphologic AS and the appearance of AS due to compromised ventricular function (termed pseudo-AS). Although the other options may aid in other facets of this patient’s management, none will allow the clinician to make this differentiation.
++
++
++
Which of the following parameters is typically reduced in chronic severe aortic regurgitation?
++
++
++
A. Diastolic blood pressure
++
++
B. Left ventricular afterload
++
++
C. Left ventricular diameter
++
++
D. Left ventricular preload
++
++
E. Total left ventricular stroke volume
+
++
The answer is A. (Chap. 287) An increase in the LV end-diastolic volume (increased preload) constitutes the major hemodynamic compensation for aortic regurgitation (AR); thus, preload is increased. The total LV stroke volume also increases to attempt to maintain effective LV stroke volume (total LV stroke volume – regurgitant volume). To do this, the LV must dilate. As the LV dilates, the wall tension to develop a given systolic blood pressure must increase as dictated by LaPlace’s law, and thus, afterload is increased. During diastole, as a large volume of blood leaves the systemic circulation to regurgitate into the LV, the diastolic pressure falls, often equilibrating with the LV pressure in severe cases. Coronary perfusion occurs primarily during diastole and depends on the gradient between aortic pressure and LV pressure through the coronaries. This explains why patients with severe AR may manifest anginal symptoms.
++
++
++
You are caring for a new admission to the cardiac intensive care unit, a 21-year-old woman with connective tissue disease. She presented with acute shortness of breath and a chest x-ray showing diffuse pulmonary edema. Physical examination revealed a soft, short early diastolic murmur at the right upper sternal border, and emergent echocardiography showed severe aortic regurgitation with an avulsed right coronary cusp. CT chest imaging showed no aortic dissection. On arrival to the coronary care unit, she is intubated and sedated. Blood pressure is 110/50 mmHg, and heart rate is 115 bpm. Her urine output is poor, and extremities are cool. The cardiothoracic surgical team is tied up in a heart transplant and won’t be able to take this patient for at least 4 more hours. What intervention is most likely to help maintain her end-organ perfusion until surgical intervention?
++
++
++
A. Esmolol continuous IV infusion titrated to a heart rate of 60–70 bpm
++
++
B. Intra-aortic balloon pump
++
++
++
++
++
++
+
++
The answer is C. (Chap. 283) This patient has severe, acute AR and indeed warrants emergent surgery. Note that her murmur is very quiet and short. In the setting of acute, severe valvular regurgitant lesions, the pressure gradient between the two chambers (in this case, the aorta and LV) quickly equilibrates during the regurgitant period. Sometimes, patients with these lesions can have no audible murmur at all, making diagnosis challenging. In this case, the goal is to reduce the regurgitant volume and thus increase effective stroke volume (total stroke volume – regurgitant volume). Interventions that increase systemic vascular resistance (vasopressin or norepinephrine) will increase regurgitant volume. Likewise, because aortic regurgitation occurs during diastole, interventions that increase diastolic time (e.g., β-blockers) will also worsen regurgitant volume. Because intra-aortic balloon pumps inflate during diastole, they will also worsen regurgitation from aorta to LV and are contraindicated in moderate or worse AR. Nitroprusside will reduce systemic vascular resistance and thus reduce the driving pressure for regurgitation. Careful administration of nitroprusside, often with concomitant invasive hemodynamic monitoring, may stabilize organ perfusion and allow surgical correction. It is important to realize that no medical therapy will correct this abnormality; surgical correction is the only definitive therapy.
++
++
++
What is the most common cause of obstruction to left ventricular inflow?
++
++
++
A. Congenital mitral valve disease
++
++
++
++
C. Infective endocarditis with large mitral valve vegetations
++
++
D. Mitral annular calcification
++
++
E. Rheumatic mitral disease
+
++
The answer is E. (Chap. 284) Rheumatic fever remains the leading cause of mitral stenosis (MS). Other less common etiologies of obstruction to LV inflow include congenital mitral valve stenosis, cor triatriatum, mitral annular calcification with extension onto the leaflets, systemic lupus erythematosus, rheumatoid arthritis, left atrial myxoma, and infective endocarditis with large vegetations. Pure or predominant MS occurs in approximately 40% of all patients with rheumatic heart disease and a history of rheumatic fever. In other patients with rheumatic heart disease, lesser degrees of MS may accompany mitral regurgitation and aortic valve disease.
++
++
++
In cases of severe mitral stenosis, which of the following parameters is typically increased?
++
++
++
++
++
++
++
C. Left ventricular diameter
++
++
D. Left ventricular end-diastolic pressure
++
++
E. Pulmonary vascular compliance
+
++
The answer is B. (Chap 284) In MS, the flow obstruction between the left ventricle and left atrium creates elevated left atrial pressure to maintain cardiac output. For example, once the effective mitral valve orifice reaches 1.5 cm2, the left atrial pressure must be >25 mmHg to maintain a normal cardiac output. LV preload, diameter, and end-diastolic pressure are normal or diminished in MS. Similarly, cardiac output is normal or diminished. Due to elevated left atrial pressures, pulmonary venous pressures rise. This causes pulmonary vascular congestion and distention and reduces pulmonary vascular compliance.
++
++
++
You are caring for a 42-year-old woman with a prior history of rheumatic fever and resultant mitral stenosis. Her valvular disease is currently moderate. You know that mitral stenosis causes an elevation in left atrial pressure, which over time can cause cardiogenic pulmonary edema and pulmonary hypertension. All of the following will result in an elevation of left atrial pressure and potential worsening of lung function EXCEPT:
++
++
++
++
++
++
++
++
++
++
++
E. Running on a treadmill
+
++
The answer is C. (Chap 284) MS represents a fixed obstructive lesion between the left atrium and ventricle. Because flow across this valve occurs in diastole, any intervention that shortens diastole will cause worsened obstruction and more elevated left atrium pressures. Thus, tachycardia induced by exercise or β-agonists is detrimental in MS. Metoprolol, a β-antagonist, will not shorten diastole. Additionally, any extra volume load or higher demand for cardiac output, as in anemia, will lead to an elevated left atrial pressure. The extra blood volume of pregnancy can be particularly poorly tolerated in patients with significant MS.
++
++
++
Mrs. Bream presents to the emergency department with acute worsening of shortness of breath. She is an 84-year-old woman with severe mitral stenosis who is scheduled for percutaneous mitral balloon valvotomy in 3 days. However, today while cooking chicken salad, she noted the onset of overwhelming weakness and dyspnea. On evaluation, she appears dyspneic and in mild distress. Oxygen saturation on room air is 91%, heart rate is 55 bpm, and blood pressure is 110/80 mmHg. She has rales to the mid-lung fields bilaterally. You note that her rhythm is irregularly irregular, and ECG confirms the new onset of atrial fibrillation. You suspect that she has a very high left atrial pressure causing pulmonary edema. All of the following therapeutic interventions will help lower her left atrial pressure EXCEPT:
++
++
++
++
++
B. Percutaneous balloon mitral valvotomy
++
++
C. Synchronized DC cardioversion
++
++
D. Transvenous pacemaker placement and pacing at a heart rate of 90 bpm
++
++
E. All of the above will help lower left atrial pressure
+
++
The answer is D. (Chap. 284) The left atrial to left ventricular pressure gradient is highly dependent on heart rate because diastolic duration is inversely related to heart rate. In the setting of MS, a long diastolic time allows more time for the left atrium to empty and thus lower pressures. Hence, sinus bradycardia is advantageous in the setting of MS, and inducing a faster heart rate via pacemaker would be harmful. The loss of atrial systole during atrial fibrillation is often poorly tolerated in MS, and regaining sinus rhythm would undoubtedly lower atrial pressure. However, the vast majority of patients with MS are unable to maintain sinus rhythm over the long term because their atria tend to be very dilated. Diuretics and percutaneous mitral valvotomy will both lower left atrial pressure.
++
++
++
A 34-year-old man with rheumatic mitral stenosis is referred to you for evaluation. He enjoys playing recreational soccer and has no limitations or symptoms. His heart rate is 65 bpm at rest. Transthoracic echocardiogram reveals a normal LV size and function, a moderately dilated left atrium, a mitral valve area of 1.7 cm2, and relatively thin noncalcified leaflets. ECG shows left atrial enlargement and sinus rhythm. On exercise stress testing, his calculated pulmonary artery systolic pressure at peak exercise is 40 mmHg. Which of the following treatment plans do you recommend?
++
++
++
A. Metoprolol 25 mg orally twice daily
++
++
B. Percutaneous mitral balloon valvotomy
++
++
C. Periodic cardiology assessments and echocardiographic monitoring
++
++
D. Sildenafil 20 mg twice daily
++
++
E. Surgical mitral valve replacement
+
++
The answer is C. (Chap. 284) This patient has asymptomatic moderate MS (severe is valve area <1.5 cm2). There is no evidence that the procedural mitral valve repair improves the prognosis of patients with slight or no functional impairment. Therefore, unless recurrent systemic embolization or severe pulmonary hypertension has occurred (pulmonary artery systolic pressures >50 mmHg at rest or >60 mmHg with exercise), valvotomy is not recommended for patients who are entirely asymptomatic and/or who have mild or moderate stenosis (mitral valve area >1.5 cm2). Given this patient’s lack of symptoms and relatively low resting heart rate, β-blockade is not warranted. Similarly, sildenafil (a pulmonary vasodilator) will not help reduce pulmonary pressures.
++
++
++
You are evaluating a 65-year-old man who has a 15-year history of nonischemic cardiomyopathy with a dilated left ventricle and an ejection fraction of 15%. Yearly echocardiograms over the past 5 years have shown severe mitral regurgitation. On optimal medical therapy, the patient has NYHA class II symptoms. Today, he specifically asks whether his valve should be fixed in order to improve his survival. You should tell him:
++
++
++
A. “If you were to have high pulmonary artery pressures or develop new atrial fibrillation, we would move forward with valve repair.”
++
++
B. “In patients like you, valve repair has never been shown to improve survival”
++
++
C. “We should consider valve surgery only if repair is possible. Replacement would not improve your survival.”
++
++
D. “While surgical methods have not shown survival benefit, percutaneous mitral valve repair has been shown to reduce mortality in patients like you.”
++
++
E. “Yes, your valve should have been fixed years ago.”
+
++
The answer is B. (Chap. 284) Mitral regurgitation (MR) occurs due to several reasons. Distinction should be drawn between primary (degenerative, organic) MR, in which the leaflets and/or chordae tendineae are primarily responsible for abnormal valve function, and functional (secondary) MR, in which the leaflets and chordae tendineae are structurally normal but the regurgitation is caused by annular enlargement, papillary muscle displacement, leaflet tethering, or their combination. This patient has functional MR due to his nonischemic dilated cardiomyopathy. In such patients, particularly those with an ejection fraction (EF) <30%, there is concern that repairing the mitral valve will lead to an overall increase in LV afterload and worsening of LV function. It is important to realize that the state of MR is one of very low afterload for the LV because it can eject easily into the low-pressure, compliant left atrium. Thus, even a slight reduction in EF (<60%) can represent significant LV dysfunction that can be unmasked after mitral valve repair. In patients with ischemic MR and significantly impaired LV systolic function (EF <30%), the risk of surgery is higher, recovery of LV performance is incomplete, and long-term survival is reduced. Referral for surgery must be individualized and made only after aggressive attempts with guideline-directed medical therapy and cardiac resynchronization therapy, when indicated. The routine performance of valve repair in patients with significant MR in the setting of severe, functional, nonischemic dilated cardiomyopathy (such as this patient) has not been shown to improve long-term survival compared with optimal medical therapy. Percutaneous therapy is currently under investigation for patients with functional MR but has not yet been shown to improve survival.
++
++
++
You are performing a right and left heart catheterization on a patient with stenosis involving two valves. Which two valves are involved based on the pressure readings listed below?
++
++
++
++
++
++
++
++
++
++
++
++
E. Pulmonic and tricuspid
+
++
The answer is D. (Chap 285) Note the gradient between right atrial pressure and right ventricular end-diastolic pressure (15 and 6 mmHg, respectively) and the left atrium (represented by the pulmonary capillary wedge pressure) and left ventricular end-diastolic pressure (12 and 6 mmHg, respectively). This is diagnostic of mixed tricuspid and mitral stenosis. Also note that the gradient between the left atrium and left ventricle is not severe and that the pulmonary arterial and right ventricular pressures are not very elevated. In the presence of tricuspid stenosis, flow is decreased to the pulmonary vasculature and left heart, thus blunting the hemodynamic effects of even severe mitral stenosis. Grading the severity of mitral stenosis in this setting is quite difficult. Finally, this mix of valvular disease is essentially diagnostic of rheumatic disease. The tricuspid valve is never involved alone and is always affected only if the mitral is also involved. Radiation injury and carcinoid syndrome are two other, rarer causes of combined mitral and tricuspid valvular disease.
++
++
++
You are evaluating a 50-year-old woman with idiopathic pulmonary arterial hypertension. Her last transthoracic echocardiogram noted severe tricuspid regurgitation in addition to a dilated hypokinetic right ventricle and estimated pulmonary artery systolic pressures exceeding 70 mmHg. On your exam, she has lower extremity edema, hepatomegaly with a pulsatile liver, jugular venous pulse elevated to the mandible with marked c-v waves and a prominent y descent, and an RV heave. She reports breathlessness with moderate exertion. What is the best treatment for her severe tricuspid regurgitation?
++
++
++
A. Diuretics and salt restriction accompanied by medical therapy targeting her elevated pulmonary artery pressures
++
++
B. Percutaneous balloon valvotomy
++
++
C. Percutaneous tricuspid valve repair
++
++
D. Surgical mitral valve replacement
++
++
E. Surgical tricuspid valve repair
+
++
The answer is A. (Chap. 285) This patient has severe tricuspid regurgitation (TR) secondary to RV dilation and dysfunction, which is in turn secondary to her severe pulmonary arterial hypertension. In cases of functional TR, repair is relegated to instances where surgery is already being pursued for left-sided valvular lesions. Furthermore, the presence of severe pulmonary hypertension is a relative contraindication for TR repair. The RV suddenly finds itself without the “pop-off valve” of severe TR and often fails when faced with the overwhelming afterload of the abnormal pulmonary vasculature. Percutaneous tricuspid valve repair is not currently performed in clinical practice. Balloon valvotomy is a repair for tricuspid stenosis, not regurgitation. There is no indication in this patient for repair of the mitral valve. Patients with mitral valvular disease may develop significant pulmonary hypertension; however, the diagnosis of idiopathic pulmonary arterial hypertension cannot be established until significant mitral valve dysfunction has been ruled out.
++
++
++
You are seeing a 21-year-old woman for the first time today in the primary care clinic. She has never seen a physician before because her parents did not believe in Western medicine. On history, she states that she feels tired occasionally and feels like she could not quite keep up with her peers in college physical education classes. On examination, you note a systolic murmur in the left second interspace preceded by a presystolic click. Transthoracic echocardiogram confirms the presence of pulmonic stenosis with a peak gradient of 60 mmHg and doming of the pulmonic valve without any pulmonic regurgitation. What is her best treatment option?
++
++
++
A. Diuretics and salt restriction
++
++
++
++
C. Percutaneous balloon valvotomy
++
++
D. Percutaneous pulmonic valve replacement
++
++
E. Surgical pulmonic valve replacement
+
++
The answer is C. (Chap. 285) Pulmonic stenosis is a rare valvular lesion encountered in adults. Fortunately, percutaneous balloon valvotomy often provides a highly efficacious, relatively low-risk therapeutic option. Diuretics can be used to treat symptoms and signs of right heart failure. Provided there is less than moderate pulmonic regurgitation, pulmonic balloon valvotomy is recommended for symptomatic patients with a domed valve and a peak gradient >50 mmHg (or mean gradient >30 mmHg) and for asymptomatic patients with a peak gradient >60 mmHg (or mean gradient >40 mmHg). Surgery may be required when the valve is dysplastic (as seen in patients with Noonan syndrome and other disorders).
++
++
++
You are taking care of a 77-year-old patient with severe aortic stenosis in the cardiac intensive care unit. Surgical aortic valve replacement is planned for tomorrow. However, suddenly, he becomes severely short of breath and manifests signs of acute pulmonary edema. On auscultation, you can now appreciate a soft, short apical systolic murmur (in addition to his previously appreciated murmur of aortic stenosis) that was not present previously. You suspect that he has suffered a ruptured mitral valve chordae and now has severe, acute mitral regurgitation. Which of the following parameters will likely increase due to his new severe mitral regurgitation?
++
++
++
++
++
B. Calculated aortic valve area
++
++
C. Effective stroke volume
++
++
++
++
E. Left ventricular afterload
+
++
The answer is D. (Chap. 286) This patient is in a precarious situation and likely will require emergent surgical intervention to survive. The combination of severe obstruction to LV outflow (severe AS) and acute, severe regurgitant mitral valve disorder will inevitably lead to intractable pulmonary edema and cardiogenic shock unless both structural abnormalities are corrected. At the advent of severe mitral regurgitation, the LV will be more effectively “unloaded” because now it can eject not only against the stenosed aortic valve, but also into the relatively low-pressure left atrium, and thus afterload will decline. Likewise, with more of the stroke volume going ineffectively into the left atrium, the effective stroke volume (total stroke volume – regurgitant volume) will decline. The aortic valve gradient will decline merely because there is less volume going across the aortic valve with each contraction. Likewise, because both the catheter-derived and echocardiographic-derived calculations of aortic valve area are dependent on the gradient for calculation, they will decline. Ejection fraction, however, will increase as the LV will be better able to contract in the state of relatively lower afterload. This highlights a misconception that higher ejection fraction is always better. LV ejection fraction is highly dependent not only on the contractile state of the ventricle, but also on its afterload state.
++
++
++
Mr. Milsap is one of your longstanding clinic patients who has a history of rheumatic heart disease. His last echocardiogram noted a mean mitral valve gradient of 11 mmHg with a calculated valve area of 1.3 cm2 at a heart rate of 60 bpm. He presents today complaining of worsening shortness of breath, and his ECG shows atrial fibrillation at a rate of 60 bpm. He has never had any bleeding episodes and had normal hematologic counts on his last check 2 weeks earlier. Which of the following options for thromboembolic prophylaxis is appropriate?
++
++
++
++
++
++
++
++
++
++
++
E. More information is needed prior to initiating thromboembolic prophylaxis.
+
++
The answer is D. (Chap. 286) Patients with valvular atrial fibrillation are at particularly high risk for systemic thromboembolisms, including stroke. In general, unless a direct contraindication is present, these patients warrant systemic anticoagulation. It is important to remember that the novel oral anticoagulants (apixaban, dabigatran, and rivaroxaban), although easier to administer, are not approved for valvular atrial fibrillation. Warfarin remains the best “tried-and-true” option for valvular atrial fibrillation.
++
++
++
All of the following are risk factors for development of peripartum cardiomyopathy EXCEPT:
++
++
++
++
++
++
++
++
++
++
++
+
++
The answer is C. (Chap. 287) Peripartum cardiomyopathy is a rare complication of pregnancy and can occur during the last trimester or within the first 6 months postpartum. Risk factors include advanced age, increased parity, twin pregnancy, malnutrition, use of tocolytic therapy for premature labor, and preeclampsia.
++
++
++
You are evaluating a new patient in clinic. The 25-year-old patient was diagnosed with “heart failure” in another state and has since relocated. He has NYHA class II symptoms and denies angina. He presents for evaluation and management. On review of systems, the patient has been wheelchair bound for many years and has severe scoliosis. He has no family history of hyperlipidemia. His physical examination is notable for bilateral lung crackles, an S3, and no cyanosis. An ECG is obtained in clinic and shows tall R waves in V1 and V2 with deep Qs in V5 and V6. An echocardiogram reports severe global left ventricular dysfunction with reduced ejection fraction. What is the most likely diagnosis?
++
++
++
A. Amyotrophic lateral sclerosis
++
++
++
++
C. Chronic thromboembolic disease
++
++
D. Duchenne muscular dystrophy
++
++
E. Ischemic cardiomyopathy
+
++
The answer is D. (Chap. 287) Cardiac involvement is common in many of the neuromuscular diseases. The ECG pattern of Duchenne muscular dystrophy is unique and consists of tall R waves in the right precordial leads with an R/S ratio >1.0, often with deep Q waves in the limb and precordial leads. These patients often have a variety of supraventricular and ventricular arrhythmias and are at risk for sudden death due to the intrinsic cardiomyopathy as well as the low ejection fraction. Implantable cardioverter defibrillators should be considered in the appropriate patient. Global left ventricular dysfunction is a common finding in dilated cardiomyopathies, whereas focal wall motion abnormalities and angina are more common if there is ischemic myocardium. This patient is at risk for venous thromboembolism; however, chronic thromboembolism would not account for the severity of the left heart failure and would present with findings consistent with pulmonary hypertension. Amyotrophic lateral sclerosis is a disease of motor neurons and does not involve the heart. This patient would be young for that diagnosis. An advanced atrial septal defect would present with cyanosis and heart failure (Eisenmenger physiology).
++
++
++
Which of the following is the most common inheritance pattern for familial cardiomyopathies?
++
++
++
A. Autosomal dominant driven by exon duplication
++
++
B. Autosomal dominant driven by missense mutations
++
++
C. Autosomal recessive driven by exon deletion
++
++
D. Autosomal recessive driven by exon duplication
++
++
E. X-linked driven by exon deletion
+
++
The answer is B. (Chap. 287) Most familial cardiomyopathies are inherited in an autosomal dominant pattern, with occasional autosomal recessive and X-linked inheritance. Missense mutations with amino acid substitutions are the most common in cardiomyopathy. Expressed mutant proteins may interfere with function of the normal allele through a dominant negative mechanism. Mutations introducing a premature stop codon (nonsense) or shift in the reading frame (frameshift) may create a truncated or unstable protein the lack of which causes cardiomyopathy (haploinsufficiency). Deletions or duplications of an entire exon or gene are uncommon causes of cardiomyopathy, except for the dystrophinopathies.
++
++
++
A 45-year-old man with a history of obesity presents to the emergency department with dyspnea, fatigue, and a nocturnal cough that has been worsening for the past several months. He denies any chest pain or pressure at rest or with exertion. On evaluation, he has evidence of cardiomegaly with a displaced PMI and elevated filling pressures with bilateral pulmonary rales and elevated jugular venous pulse. Echocardiography reveals a globally depressed left ventricular ejection fraction of 25% with a dilated left ventricle. Which of the following tests is a Level I recommendation for further workup?
++
++
++
++
++
++
++
C. Erythrocyte sedimentation rate
++
++
D. Serum iron and transferrin saturation
++
++
E. Thyroid-stimulating hormone level
+
++
The answer is E. (Chap. 287) All of the tests listed have a role in some patients with newly diagnosed cardiomyopathy. However, only a thyroid-stimulating hormone (TSH) level receives a Level I recommendation from the American College of Cardiology (ACC)/American Heart Association (AHA) for all patients with new cardiomyopathy. All other tests listed are driven by symptoms and signs that are present. Cardiac MRI and erythrocyte sedimentation rate (ESR) levels may help diagnose inflammation in patients who are suspected to have an acute myocarditis or infiltrative disease. Coronary angiography is only recommended for patients with angina, although one must be careful to include dyspnea on exertion as an angina equivalent. Serum iron and transferrin saturation are worth checking in patients suspected of hemochromatosis. Table V-100 lists tests available for the initial evaluation of new cardiomyopathy and indicates which receive a Level I recommendation for all patients.
++
++
++
++
A 22-year-old college student with no past medical history was seen in the urgent care clinic 3 days ago for coryza, myalgias, cough, and fever, which was typical of the viral upper respiratory illness making its way through the campus. He was given a cough suppressant and antipyretics and advised to remain hydrated. Today, he presents to the emergency department with lethargy and fatigue. He is obtunded with a heart rate of 120 bpm and blood pressure of 78/62 mmHg. His extremities are cool, and jugular venous pulse is elevated nearly to the mandible. Precordial auscultation reveals very quiet heart sounds, an S3 gallop, and a soft murmur of mitral regurgitation. Emergent transthoracic echocardiogram shows no pericardial effusion, a nondilated left ventricle with an ejection fraction of 30%, and mild mitral regurgitation. Endomyocardial biopsy shows lymphocytic myocarditis. Which of the following statements regarding this patient’s prognosis and implications for therapy are true?
++
++
++
A. His chance of survival is <10% without cardiac transplantation. Emergent transplant listing is warranted.
++
++
B. His chance of survival is >50%, with many similar patients having a full recovery in left ventricular function over the ensuing weeks to months. Aggressive pharmacologic and mechanical hemodynamic support is warranted.
++
++
C. Immunosuppression with high-dose systemic steroids will increase his chance of survival.
++
++
D. The presence and titer of anti-heart antibodies can help provide prognostic information for this patient.
+
++
The answer is A. (Chap. 287) This patient presents with a classic history for fulminant viral myocarditis. A small number of patients present with fulminant myocarditis, with rapid progression from a severe febrile respiratory syndrome to cardiogenic shock that may involve multiple organ systems, leading to renal failure, hepatic failure, and coagulopathy. These patients are typically young adults who have recently been dismissed from urgent care settings with antibiotics for bronchitis or oseltamivir for viral syndromes, only to return within a few days in rapidly progressive cardiogenic shock. Prompt triage is vital to provide aggressive support with high-dose intravenous catecholamine therapy and sometimes with temporary mechanical circulatory support. Recognition of patients with this fulminant presentation is potentially lifesaving because more than half can survive, with marked improvement demonstrable within the first few weeks. The ejection fraction function of these patients often recovers to near-normal, although residual diastolic dysfunction may limit vigorous exercise for some survivors. There is no established role for measuring circulating anti-heart antibodies, which may be the result, rather than a cause, of myocardial injury and have also been found in patients with coronary artery disease and genetic cardiomyopathy. There is currently no specific therapy recommended during any stage of viral myocarditis. Large trials of immunosuppressive therapy for Dallas Criteria–positive myocarditis have been negative.
++
++